FCM Cases Flashcards

1
Q

A 34-year-old woman who has no past medical problems nor is currently taking any medications comes into your office because she noticed a tender lump in her left breast starting approximately one month ago. She is worried because she has an aunt who had breast cancer that was BRCA positive, though her mother is BRCA negative. Her periods have been regular since they started at the age of 13 and occur every 32 days. She is currently menstruating. She has three children aged 12, 9, and 4. On exam, her BMI is 32, up from 28 three years ago and her other vital signs are stable. On breast exam, you note a mobile rubbery mass of approximately 1 x 1cm and with regular borders that is tender to palpation. You appreciate no axillary adenopathy. The rest of her physical exam is unremarkable. Of the information provided, which of the following places this patient at increased risk for breast cancer?

A. Age
B. Weight
C. Parity history
D. Family history of cancer
E. Age of menarche
A

The correct answer is B.
With a BMI of 32, obesity is the one risk factor for this patient based on the information given. Other risk factors for breast cancer include family history of breast cancer in a first degree relative (mother or sister - not aunt), prolonged estrogen exposure (menarche before age 12, menopause after 45, advanced age at first pregnancy), genetic predisposition (BRCA 1 or 2 mutation), advanced age (breast cancer risk increases with age, and this patient is relatively young), female sex, increased breast density and certain exposures (diethylstilbestrol, hormone or radiation therapy, heavy smoking).

How well did you know this?
1
Not at all
2
3
4
5
Perfectly
2
Q

A 64-year-old woman who is overweight with well-controlled hypertension comes to your office complaining of a lump in her breast that she noticed while showering. She denies any pain, tenderness, or skin changes. A pertinent review of systems is negative. Menarche began at the age of 10. Her first child was born when she was 31 and she had her second and last child at the age of 33. She experienced menopause at the age of 44. Her mother died of colon cancer when she was 65 and her father passed away from metastatic prostate cancer at the age of 70. She has no history of tobacco use ever and occasionally drinks a glass of wine with dinner. Her BMI is 34. Which of the information provided thus far puts the patient at decreased risk for breast cancer?

A. Age
B. Weight
C. Age at first birth
D. Age at menarche
E. Age at menopause
A

The correct answer is E.
The patient experienced menopause at the age of 44, which shortens her time of estrogen exposure, a known risk factor for the development of breast cancer. Factors associated with decreased breast cancer risk include pregnancy at an early age, late menarche, early menopause, high parity and medications such as selective estrogen receptor modulators along with NSAIDs and aspirin. Risk factors for breast cancer include family history of breast cancer in a first degree relative (mother or sister), prolonged estrogen exposure (menarche before age 12, menopause after 55, advanced age at pregnancy, obesity after menopause), female sex, genetic predisposition (BRCA 1 or 2 mutation), advanced age (breast cancer risk increases with age), increased breast density and exposures (diethylstilbestrol, hormone or radiation therapy, smoking).

How well did you know this?
1
Not at all
2
3
4
5
Perfectly
3
Q

A 63-year old woman comes into your office for her annual preventive exam. She has hypertension and type 2 diabetes. She is not sexually active. Her blood pressure is 125/80 and her physical exam otherwise is within normal limits. You recommend influenza and zoster vaccination. Her last colonoscopy was eight years ago and her last mammogram one year ago was normal. She has never had an abnormal Pap smear. At the age of 45 she had a total hysterectomy for fibroids. You tell her she does not require a Pap smear today because:

A. She has never had an abnormal Pap smear
B. She is not sexually active
C. She had a total hysterectomy for fibroids
D. She is 63 years old
E. She experienced menopause more than 10 years ago

A

The correct answer is C.
The patient described above underwent a total hysterectomy (total removal of the uterus and cervix with or without oophorectomy) for benign reasons (fibroids). USPSTF guidelines recommend against continued cervical cancer screening in patients whose uterus has been removed for benign disease and evidence showed cytologic screening to be very low yield and poor evidence that screening to detect vaginal cancers improves health outcomes in women after hysterectomy for benign disease. Cervical cancer screening should begin at the age of 21 and women between the ages of 65 and 70 who have had three or more normal Pap tests in the past ten years may choose to stop cervical cancer screening. Not being sexually active; age 63; only having had normal PAP smears and years since menopause are not reasons to stop screening for cervical cancer.

How well did you know this?
1
Not at all
2
3
4
5
Perfectly
4
Q

A 47-year-old woman comes into your office for a health care maintenance exam. She has hypertension and type 2 diabetes. She is not sexually active and has not yet experienced menopause. There is no family history of cancer. Her blood pressure is 118/78, her BMI is 34 and the remainder of her physical exam is within normal limits. Her vaccinations are up-to-date, she has a PAP smear today and will have labs drawn. According to USPSTF, which of the following is the best recommendation to give her concerning mammography?

A. Should have started at age 40 and every year thereafter
B. Should have started at age 40 and every 2 years thereafter
C. Start at age 50 and every year thereafter
D. Start at age 50 and every 2 years thereafter
E. Should have started at age 45 and every year thereafter

A

The correct answer is D.
Mammography has a sensitivity of 60% to 90% for detecting breast cancer and decreases breast cancer mortality. According to the most recent USPSTF guidelines, routine mammography is not indicated for women younger than 50 years old except as based on patient context (history) and beliefs about risks/benefits. The USPSTF recommends biennial testing for women between the ages of 50 and 74. There is insufficient evidence to assess the benefits versus risk of screenings in women after the age of 75. Other groups such as the American Cancer Society (ACS) and American College of Obstetricians and Gynecologists (ACOG) recommend yearly mammograms starting at age 40, continuing as long as the woman is in good health.

How well did you know this?
1
Not at all
2
3
4
5
Perfectly
5
Q

A 27-year-old woman comes into your office because she heard from her friend about a vaccination against cervical cancer and would like one. She has no medical problems and has had a Mirena IUD for three years. She has an allergy to latex and penicillin. She began having sex at the age of 18 and is currently sexually active with one partner. She occasionally uses condoms. She smokes half a pack of cigarettes per day. Her mother had endometrial cancer several years ago and had a total hysterectomy. Why is the patient not an ideal candidate for the Gardasil vaccination?

A. Age
B. Sexual activity
C. Mirena IUD
D. Allergy to penicillin
E. Tobacco use
A

The correct answer is A.
Gardasil9 is a vaccination against 9 HPV types and is approved for females ages 9 to 26. While the recommendation is to end at age 26, that does not mean it is dangerous; it just hasn’t been studied and will not likely be covered by insurance. Activity (B) is not a contraindication to Gardasil vaccination. The other choices are not contraindications to vaccinations.

How well did you know this?
1
Not at all
2
3
4
5
Perfectly
6
Q

A 55-year-old man with no significant past medical history presents for a routine physical exam. He last saw a doctor five years ago. Social history is remarkable for a 35-pack-year tobacco history since the age of 20. He indicates that his wife and children have urged him to quit smoking for the last few months. When you ask him if he has considered quitting, he replies, “I just don’t see what the big deal is!” Which stage of change best describes this patient at this time?

A. Precontemplation
B. Contemplation
C. Preparation
D. Action
E. Maintenance
A

The correct answer is A.
Based on this man’s response, it appears he has not actively considered quitting smoking despite his family’s concern. All options refer to different stages in the Transtheoretical stages of change model. Given that he has not actively contemplated quitting, the best stage to describe this patient at this time would be the Precontemplation stage and not any of the other responses.

How well did you know this?
1
Not at all
2
3
4
5
Perfectly
7
Q

A 48-year-old man with a past medical history that includes hypertension, Chronic Obstructive Pulmonary Disease (COPD), and hyperlipidemia presents to clinic as a new patient for a general physical exam. History reveals that he has been smoking a pack of cigarettes daily since age 20. He drinks two beers daily. He is intermittently noncompliant with his medications. Review of the state immunization database reveals that the only immunization he has received as an adult was a tetanus diphtheria shot administered 12 years ago. Which of the following vaccine combinations would be most appropriate for this patient?

A. Influenza, Meningococcal, and Zoster
B. Influenza, Pneumococcal, and Tdap
C. Influenza, Zoster, and Tdap
D. Meningococcal, Pneumococcal, and Tdap
E. Meningococcal, Pneumococcal, and Zoster
A

The correct answer is B.
Because this man has a diagnosis of COPD and smokes cigarettes, both annual Influenza and Pneumococcal vaccination are indicated. Because his last tetanus immunization was over 10 years ago and because he has not had a booster pertussis shot as an adult, a one-time Tdap is recommended. At this time meningococcal vaccine is recommended for adolescents and young adults and not indicated for this patient. Zoster vaccine is recommended to all adults at age 50 or older.

How well did you know this?
1
Not at all
2
3
4
5
Perfectly
8
Q

A 55-year-old man comes to the clinic for a visit. He has read about the dangers of being overweight and inquires about which category he fits into. He is 5’ 10’’ (1.78 m) and weighs 220 lbs (100 kg), BMI = 31.6. Which of the following categories most accurately describes the patient based on his BMI?

A. Underweight
B. Ideal
C. Overweight
D. Obese
E. Morbidly (very severely) obese
A

The correct answer is D.
Based on BMI measurements, Underweight is considered < 18.5; Ideal: 18.5 to 25; Overweight 25 to 30; Obese 30 to 40; Morbidly (very severely) obese > 40.

How well did you know this?
1
Not at all
2
3
4
5
Perfectly
9
Q

A 55-year-old man with a family history of melanoma presents to the clinic for evaluation of a skin lesion on his back which appeared three months ago. His wife first alerted him to it, hasn’t noticed it change and he has not noticed any symptoms associated with it. Physical examination reveals a 7 mm uniformly black macule that is symmetrically round with sharply demarcated borders on his upper back near the right shoulder. Which of the following characteristics would most justify it being biopsied today?

A. Symmetry
B. Borders
C. Color
D. Diameter
E. Location
A

The correct answer is D.
Using the ABCDE mnemonic, this nevus is not Asymmetrical, does not have irregular Borders, does not display Color variation and he does not describe any Evolution or change or symptoms. The only positive is that its Diameter is > 6 mm, which is considered a red flag supporting biopsy. Location is not considered a predictive factor for melanoma.

How well did you know this?
1
Not at all
2
3
4
5
Perfectly
10
Q

A 55-year-old man with no significant past medical history and generally healthy behaviors presents to clinic for a health care maintenance exam. He says, “I’d like to get tested for all types of cancer.” He does not have any family history of cancer. Review of systems is negative for any symptoms of prostate cancer, such as urinary frequency, urgency, retention, hematuria, weight loss, or back pain. He is a lifelong non-smoker, and he doesn’t drink alcohol or use recreational drugs. Which of the following screening tests is given either an A or B recommendation in favor of its routine use for patients such as this one?

A. Prostate Specific Antigen (PSA) testing
B. Lung cancer screening
C. Pancreatic cancer screening
D. ECG screening for coronary artery disease
E. Colon cancer screening

A

The correct answer is E.
The USPSTF gives colon cancer screening an A recommendation for people age 50 to 75 years due to clear evidence of benefit. Lung cancer screening is given a B recommendation for 55-year-old men with a 30 pack-year tobacco history and who have smoked in the past 15 years. This patient is a non-smoker. Pancreatic cancer screening and ECG screening are both given D recommendations (against their use). PSA screening is given a C recommendation, indicating that doctors and patients should make individualized decisions about the use of this test.

How well did you know this?
1
Not at all
2
3
4
5
Perfectly
11
Q

Ms. Marcos is a 65-year old woman with a past medical history of Type 2 diabetes, hypertension, and hypercholesterolemia who presents with six months of insomnia despite self-medication with acetaminophen, diphenhydramine, and herbal remedies. She is 5’ 2” and weighs 250 lbs.
When considering a differential diagnosis, which one of the following is a common cause of insomnia in the elderly?

A. Sleep Apnea
B. Pneumonia
C. Chronic sinusitis
D. Asymptomatic coronary artery disease
E. Hypoparathyroidism
A

The correct answer is A, sleep apnea.
Sleep apnea occurs in 20% to 70% of elderly patients. Obstruction of breathing results in frequent arousal that the patient is typically not aware of; however, a bed partner or family member may report loud snoring or cessation of breathing during sleep.
Some of the other most common causes of insomnia in the elderly are:
Environmental problems such as noise or uncomfortable bedding which are not conducive to sleep.
Drugs, Alcohol, and Caffeine such as over-the-counter, alternative, and certain recreational drugs.
Parasomnias such as restless leg syndrome/periodic leg movements/REM sleep behavior disorder. In restless leg syndrome, the patient experiences an irresistible urge to move the legs, often accompanied by uncomfortable sensations. In periodic leg movement and REM sleep behavior disorder, the patient experiences involuntary leg movements while falling asleep and during sleep respectively.
Disturbances in the sleep-wake cycle such as jet lag or shift work.
Psychiatric disorders such as primary depression and anxiety
Symptomatic cardiorespiratory disease (asthma, COPD, heart failure)
Pain or pruritus
Gastroesophageal reflux disease (GERD) due to heartburn, throat pain or breathing problems.
Hyperthyroidism The elderly frequently do not present with typical symptoms such tachycardia or weight loss, and therefore further laboratory studies may be required to detect this problem.

How well did you know this?
1
Not at all
2
3
4
5
Perfectly
12
Q

Ms. Anderson is a 60-year-old woman who comes in to clinic as a walk in appointment. She is tearful and is carrying a box of tissues in her hand. She says she doesn’t know why but she has been very sad of late. She reports trouble falling asleep and staying asleep. She used to be the head of her Bridge club, but quit two weeks ago and doesn’t feel like going out anymore. She also says she has lost interest in walking her dog, and now just allows him to use the doggie door to let himself out. She also says she feels weak and fatigued and no longer has the energy to do her gardening or shopping. She spends most of her day on the sofa crying while watching TV. She also reports a greatly diminished appetite. She denies suicidal or homicidal ideation, but she does have a history of a previous suicide attempt following her divorce seven years ago for which she was hospitalized. A recent CBC, CMP, CXR, TSH, U/A and CT of the head were all within normal limits. How long do the above symptoms need to be present in order to make the diagnosis of Major Depressive Disorder?

A. One week
B. Two weeks
C. Four weeks
D. Five weeks
E. Eight weeks
A

Correct Answer: B
Depressed mood or anhedonia and at least five of the following eight criteria must have been present for two weeks or longer. (Mneumonic = SIG E CAPS)
Sleep: Insomnia or hypersomnia nearly every day
Interest (loss of): Anhedonia (loss of interest or enjoyment) in usual activities
Guilt: Feelings of worthlessness or excessive or inappropriate guilt (which may be delusional) nearly every day (not merely self-reproach or guilt about being sick)
Energy (decreased): Fatigue or loss of energy nearly every day
Concentration (decreased): Diminished ability to think or concentrate, or indecisiveness, nearly every day (either by subjective account or as observed by others)
Appetite (increased or decreased)
Psychomotor agitation or retardation nearly every day (observable by others, not merely subjective feelings of restlessness or being slowed down)
Suicidal ideation: Recurrent thoughts of death (not just fear of dying), recurrent suicidal ideation without a specific plan, or a suicide attempt or a specific plan for committing suicide

How well did you know this?
1
Not at all
2
3
4
5
Perfectly
13
Q

Mr. Jones is an 82-year-old man who presents to the office for his six-month chronic disease visit. His diabetes and hypertension are controlled on his usual home medications. He reports that his wife died four weeks ago, and he is now experiencing insomnia most days of the week and fatigue and loss of energy nearly every day; reports decreased enjoyment of his activities, such as playing chess with his neighbor; and is also experiencing loss of appetite but no weight loss. He denies any suicidal ideation and has no previous suicide attempts. Mr. Jones says he often hears his wife’s voice while going to bed. He says he goes to church to pray. You are trying to determine if your patient’s symptoms are normal grief or if you should diagnose and treat him for Major Depressive Disorder (MDD). Which feature of Mr. Jones’ case would suggest MDD rather than a normal grief reaction?

A. Insomnia
B. Change in appetite
C. Inability to experience any joy
D. Hearing wife's voice
E. Fatigue
A

Correct Answer: C
The loss of a loved one can be a traumatic event and it is normal to experience a period of grief. DSM-5 states that MDD can be diagnosed during a period of grief as long as the criteria are met. Grief can be difficult to distinguish from major depression with symptoms of sadness, fatigue, changes in appetite, sleep disruption, and decreased concentration. Since your patient is also exhibiting diminished pleasure with normally enjoyable activities, this may indicate the patient has MDD, as pervasive unhappiness and misery are rarely a part of the normal grieving process. Other features that differentiate MDD from grief include:
Guilt about things other than actions taken or not taken at the time of death
Thoughts of death other than feeling that he or she would be better off dead or should have died with the deceased person
Morbid preoccupation with worthlessness
Marked psychomotor retardation
Prolonged and marked functional impairment
Hallucinatory experiences other than hearing the voice of, or transiently seeing the image of, the deceased person

How well did you know this?
1
Not at all
2
3
4
5
Perfectly
14
Q

Ms. Rogers is a 75-year-old woman who was found unresponsive in her house by her neighbor who had come over to help clean her house. An empty unlabeled pill container was found next to her on the bathroom floor. She was rushed to the ER, stabilized and is now in ICU on a mechanical ventilator. Which of the following are true regarding suicide in the elderly?

A. Elderly persons attempting suicide are more likely to be married and living with their spouse.
B. Elderly persons attempting suicide usually report good sleeping habits.
C. Suicidal behaviors increase with age, but rates of completed suicides dont.
D. Approximately 75% of the elderly who commit suicide had visited a primary care physician within the preceding month, but their symptoms went unrecognized.
E. Firearms are the most common means of suicide in the elderly.

A

Correct Answer: D
The USPSTF recommends screening all adults for depression, but especially patients with chronic diseases like diabetes, as they are at high risk for depression. The PHQ-2 inquires about the frequency of depressed mood and anhedonia over the past 2 weeks. The purpose of the PHQ -2 is not to establish a final diagnosis, but rather to screen for depression as a “first-step” approach. Patients who screen positive should be evaluated by the PHQ-9 to determine whether they meet the criteria for depression. Another screening tool which can be used is the Geriatric Depression Scale - Short Form (GDS-SF) which includes a series of 15 questions. Specifically related to suicide in the geriatric population: Elderly persons attempting suicide are more likely to be WIDOW(ER)S, AND LIVE ALONE; Elderly persons attempting suicide have REDUCED sleep quality; Suicidal behaviors DO NOT increase with age, but rates of completed suicides DO.
Drug overdose is the most common means of suicide in the elderly.

How well did you know this?
1
Not at all
2
3
4
5
Perfectly
15
Q

Ms. Burton is a 45-year-old woman who has never been to a primary care provider. She presents today to establish care and get her health in order. Her concerns today are: fatigue, weakness, numbness, insomnia, feeling sad at times, anhedonia, increased appetite, weight gain, dry skin, and increasing hair loss within the past month.

Her vital signs are:

  • Heart rate: 78 beats/minute
  • Respiratory rate: 18 breaths/minute
  • Oxygen saturation: 95%
  • Blood pressure: 152/84 mmHg
  • Weight: 325 lbs
  • Body Mass Index: 41 kg/m2

Today, her physical exam is significant for thinning hair, poor dentition, a systolic murmur heard at the left upper sternal border, an obese abdomen, and bilateral knee stiffness and pain on range of motion exam. Remainder of the physical exam is within normal limits. Which laboratory tests or studies can be done to rule out medical causes of insomnia, fatigue, and depression?

A. Chest-X Ray
B. CBC, CMP, and TSH
C. HgbA1c, lipid panel, urine microalbumin
D. CT head without contrast
E. MRI brain with contrast
A

Correct Answer: B
CMP can be used to detect electrolyte, renal and hepatic problems. TSH can be used to rule out hypo- or hyperthyroidism. CBC can be helpful to detect anemia and vitamin deficiencies. In addition, ESR can be used to test for rheumatologic disease. An ECG should be done if the patient is using drugs that might alter cardiac conductivity, such as TCAs.

How well did you know this?
1
Not at all
2
3
4
5
Perfectly
16
Q

A 60-year-old woman presents to the office complaining of increased frequency of urination and fatigue for the past several months. She denies fevers, dysuria, back pain, diarrhea and abdominal pain. She has noted some weight loss without working on diet or exercise. Her past medical history is significant for hyperlipidemia and hypertension, for which she takes simvastatin and lisinopril. She is a non-smoker and consumes one to two glasses of wine per week. Her vitals are:
Heart rate: 70 beats/minute
Blood pressure: 130/70 mmHg
Body Mass Index: 30 kg/m2
Physical examination reveals increased pigmentation in her axilla bilaterally. Her labs are as follows:
Random plasma blood glucose: 205 mg/dL
Creatinine: 0.8 mg/dL
TSH: 2.1 U/L.
What test is needed to diagnose diabetes mellitus?

A. The random blood glucose is sufficient
B. Fasting blood glucose
C. An oral glucose tolerance test
D. HgbA1c
E. Urine microalbumin
A

The correct answer is A.
The correct answer is (A). Diabetes can be diagnosed with either an HbA1c > 6.5%, a fasting plasma glucose ≥ 126 mg/dl (7.0 mmol/l), a plasma glucose ≥ 200 mg/dL (11/1 mmol/l) two hours after a 75 g glucose load, or symptoms (such as polyuria, polydipsia, unexplained weight loss) and a random plasma glucose ≥ 200 mg/dL (11.1 mmol/l).
Answers (B), (C), (D) and (E) are incorrect, as the diagnosis of diabetes can be made based on random blood glucose with symptoms.

How well did you know this?
1
Not at all
2
3
4
5
Perfectly
17
Q

A 42-year-old woman presents for a visit after recently being diagnosed with Type 2 diabetes. She has made a plan to work on diet and exercise. Her A1c is found to be 8.0%. What is the best medicine to start at this time?

A. A sulfonylurea
B. Basal insulin
C. Metformin
D. GLP-1 receptor agonist
E. An SGLT2 inhibitor
A

The correct answer is C.
The correct answer is (C). Sulfanylureas, GLP-1 receptor agonists, and SGLT2 inhibitors may be used as second-line agents. Insulin is generally not used until two other oral medications are insufficient to control the blood sugar, but most people with Type 2 diabetes become insulinopenic over time and require insulin treatment.

How well did you know this?
1
Not at all
2
3
4
5
Perfectly
18
Q

A 72-year-old woman with a 30-year history of Type 2 diabetes returns to your office for routine visit. She is taking 20 units of insulin glargine every morning and five units of insulin aspart with meals. The patient notes blurry vision for the past several months and a few days of dark spots in her vision. She denies headaches or nausea. What is true regarding diabetic retinopathy?

A. The majority of people with diabetes only develop retinopathy after 10 years with the diagnosis.
B. 40% of people with severe diabetes requiring insulin have retinopathy five years after diagnosis.
C. Vision changes are an early sign of retinopathy.
D. Primary care physicians should examine the retina on every visit for ongoing diabetes care.

A

The correct answer is B.
The correct answer is (B). The patient’s symptoms describe diabetic retinopathy. Proliferative retinopathy is prevalent in 25% of the diabetes population with ≥ 25 years of diabetes, but many patients have retinopathy much earlier. Early changes of retinopathy are asymptomatic. Patients need to see an ophthalmologist regularly for a dilated retina exam, not rely on a view of the retina from primary care physicians. Abnormalities seen include macular edema ( a common cause of blurry vision) and new blood vessel formation which can leak and cause dark spots in the vision. Diabetic eye disease often can be treated before vision loss occurs. Glaucoma (causing increased intraocular pressure) is 40% more likely in people with diabetes, but usually causes nausea, headaches, and narrowing of vision or halos around lights.

How well did you know this?
1
Not at all
2
3
4
5
Perfectly
19
Q

A 65-year-old male with known Type 2 diabetes mellitus presents to the Emergency Department with altered mental status. The patient experienced no known head trauma. His vitals are:
Temperature: 38.1 Celsius
Heart rate: 102 beats/minute
Respiratory rate: 16 breaths/minute
Blood pressure: 90/74 mmHg
His mucous membranes appear very dry and he is started on IV fluids. Neurological exam reveals no focal deficits. His plasma glucose is found to be 700 mg/dL. Urinalysis reveals no ketone bodies. What is the most likely diagnosis?

A. Thiamine deficiency
B. Diabetic ketoacidosis (DKA)
C. Cerebrovascular accident
D. Hyperosmolar hyperglycemic state (HHS)
E. Cardiac arrhythmia
A

The correct answer is D.
The correct answer is (D). HHS is seen typically in patients with Type 2 diabetes. It includes very high sugars > 600; ph > 6.4; dehydration; and lack of ketones in the urine and blood. Diabetic ketoacidosis is more common in Type 1 diabetes, and the patient will have ketone bodies in the urine. Thiamine deficiency can cause Korsakoff syndrome, and is typically seen in alcoholics with severe malnutrition, however, this patient is not a known alcoholic and doesn’t appear malnourished. Despite the confusion in this patient, stroke is an unlikely diagnosis in this case given the lack of focal deficits on exam. Cardiac arrhythmia can cause dizziness, but is less likely to cause prolonged altered mental status.

How well did you know this?
1
Not at all
2
3
4
5
Perfectly
20
Q

A 61-year-old female has recently been diagnosed with Type 2 diabetes. Her fasting glucose was 240 mg/dL and her A1c was 8.9%. Her BP has been 148/90 and 146/86 at two separate office visits. Her home BP measurements have been in a similar range. Her creatinine is 0.9 and she has no known heart disease. She currently takes losartan 100 mg daily for a diagnosis of hypertension. Which of the following would be the most appropriate step in managing this patient’s blood pressure?

A. Make no changes to her medications as her blood pressure is at goal.
B. Start lisinopril daily.
C. Start amlodipine daily.
D. Start metoprolol daily.
E. Start furosemide daily.
A

The correct answer is C.
The correct answer is (C). According to the 2017 AHA/ACC blood pressure guidelines, this patient’s blood pressure goal should be 130/80 mmHg. She is clearly above that, and she should have a blood pressure medication added (or in a highly motivated patient, dramatic behavioral changes with close follow up). There is no preference for a first line treatment for blood pressure in diabetic patients, although many providers start with and ACE inhibitor or ARB because diabetes is a risk factor for chronic kidney disease. Furosemide and metoprolol are not among the four major classes of medications for blood pressure management (ACEIs, ARBS, calcium channel blockers, and thiazides), so D and E are not acceptable choices. Lisinopril is an acceptable first-line choice, but it should not be combined with an ARB. Since this patient is taking losartan (an ARB), adding an ACEI is contraindicated. Amlodipine is a good choice for this patient.

How well did you know this?
1
Not at all
2
3
4
5
Perfectly
21
Q

A 65-year-old female presents to your office for a routine visit. She is found to have a blood pressure of 146/96 mmHg. You repeat the blood pressure in her other arm and get 148/92 mmHg. Her pulse is 70 and regular. Her last BP reading was one year ago and was 120/76 mmHg. She has no other medical problems. Her BMI is 28. She states that she likes to walk 30 minutes every other day with her husband and has been doing that for years now. At this time, the most appropriate diagnosis is…

A. White coat hypertension
B. Elevated blood pressure reading
C. Stage 1 hypertension
D. Stage 2 hypertension
E. Secondary hypertension
A

The correct answer is B.
To diagnose hypertension, two separate readings greater than 130/80 mmHg each time - taken a week or more apart - are needed. Furthermore, ideally home blood pressure readings in the hypertensive range would be needed to confirm that she does not have white coat hypertension. Because this patient has had elevated blood pressure documented on only one occasion (today), the most appropriate current diagnosis is elevated blood pressure.
If she has a second similarly elevated reading, Stage 2 hypertension may be diagnosed.
Stage 1 hypertension refers to blood pressures between 130-139/80-89 mmHg.
This patient has not yet been diagnosed with hypertension, so neither A, C nor D is appropriate.

How well did you know this?
1
Not at all
2
3
4
5
Perfectly
22
Q

A 68-year-old male was diagnosed with Stage 1 essential hypertension a few months ago and has been working on diet and lifestyle modifications. He has a BMI of 28, mild knee arthritis but no other medical diagnoses. He has been a patient of yours for several years, and returns today as planned. Today his blood pressure is 156/94 mmHg. The remainder of his cardiovascular exam is within normal limits. After counseling the patient, he agrees to start an antihypertensive medication. His creatinine is 0.9, urinalysis is normal, and electrolytes are within normal limits. which of the following is the most appropriate medication to begin in this patient?

A. Beta blocker
B. Thiazide diuretic
C. Nitrate
D. Loop diuretic
E. Clonidine
A

The correct answer is B.
This patient now meets criteria for Stage 2 hypertension as indicated by a systolic BP 140 to 159 mmHg and diastolic BP 90 to 99 mmHg.
The ACC/AHA guidelines recommend thiazide diuretics, ACE inhibitors, angiotensin II receptor blockers, or calcium channel blockers as first-line treatment for most patients with newly diagnosed hypertension, with a slight preference for chlorthalidone in the diuretic class.
The other options are not first-line treatments for hypertension.

How well did you know this?
1
Not at all
2
3
4
5
Perfectly
23
Q

A 54-year-old male with a history of chronic gout and GERD presents to your office for his health maintenance exam. Vitals today are blood pressure 138/88 mmHg, pulse 65 beats/min, respirations 10/min, afebrile, BMI 29 kg/m2. He smokes 10 cigarettes per day, does not regularly exercise, and drinks one to two beers daily, four or five times a week. He has no current concerns, review of systems is negative, and his physical exam is unremarkable. You recommend lifestyle changes. Which of the following changes is least likely to improve his blood pressure?

A. Smoking cessation
B. DASH eating plan
C. Weight loss
D. Alcohol cessation
E. Increased exercise
A

The correct answer is D.
While all of these options are reasonable lifestyle modifications to recommend for patients, this particular patient’s blood pressure is least likely to be reduced by alcohol cessation. Moderate alcohol consumption actually improves blood pressure by 2 to 4 mmHg; therefore, stopping his moderate alcohol consumption could increase his blood pressure. However, it is not recommended to encourage alcohol use in patients who do not drink, because of the risk of encouraging problem drinking.
Initiation of the NIH-sponsored Dietary Approaches to Stop Hypertension (DASH) eating plan has been shown to lower systolic pressure as have smoking cessation, weight loss, and increased exercise.

How well did you know this?
1
Not at all
2
3
4
5
Perfectly
24
Q

A 60-year-old male with a past medical history of chronic gout, depression, and Stage 1 hypertension presents to your office for a follow-up visit. He has been attempting to reduce his blood pressure with behavioral changes, but has had difficulty maintaining the changes. Today, his vitals are blood pressure 144/90 mmHg, pulse 78 beats/min, respirations 12/min, temperature 98.7 F. His recent basic metabolic panel was completely normal. Based on cholesterol levels he had done in the prior week, you calculate his 10-year ASCVD risk at 11%. As you consider starting a medication for his hypertension, which of the following medications is most likely to cause an adverse event in this patient?

A. Lisinopril
B. Hydrochlorothiazide
C. Amlodipine
D. Losartan
E. Metoprolol
A

The correct answer is B.
Hydrochlorothiazide (HCTZ) can cause hyperuricemia and therefore should be used with caution in patients with gout.
Metoprolol is not a first-line choice for the management of blood pressure, but there is no particular reason to expect this patient to experience an adverse drug event due to a beta-blocker.
While all of the other medications listed are appropriate first-line anti-hypertensives, many clinicians would select one of the other options over HCTZ for this patient given his history of gout.

How well did you know this?
1
Not at all
2
3
4
5
Perfectly
25
Q

A 62-year-old woman presents for follow-up of her hypertension and diabetes. In general, her chronic diseases are well controlled and she has suffered no target organ damage. She has worked hard to begin exercising, and is walking vigorously five times a week. She has also worked hard on dietary changes, and has been following the DASH eating plan very seriously. She quit smoking three months ago. Her blood pressure today is 148/88 mmHg, pulse is 72 and BMI is 32. She is taking metformin 500 mg twice daily, simvastatin 20 mg daily and hydrochlorothiazide (HCTZ) 25 mg daily, and she is compliant with her daily medications. Her labs today include an A1C of 6.6, an LDL of 88 and a basic metabolic panel within normal limits. Which of the following management steps today do you consider the most appropriate?

A. Increase HCTZ to 50 mg daily
B. Make no changes as she is at her treatment goals
C. Impress upon her the importance of making more lifestyle modifications
D. Add amlodipine 5 mg daily
E. Change her simvastatin to atorvastatin 20 mg

A

The correct answer is D.
The goal blood pressure for patients with hypertension is 130/80 mmHg, and this patient has not met this goal with HCTZ and major lifestyle changes.
Increasing the dose of HCTZ from 25 to 50 does not improve blood pressure further, so adding a second medication would be more beneficial. While commending her on her lifestyle changes is important, counseling about intensifying them is not likely to be realistic nor helpful given all that she has already done.
There is no need to change her statin, however calculating her ASCVD risk to determine whether she is on the appropriate dose would be helpful.
The current cholesterol guidelines recommend a moderate intensity statin for patients with diabetes. For simvastatin, a dose of 10 mg represents a low-intensity dose. Increasing this to 20 mg would put her management more in line with these guidelines, though it would not address her elevated blood pressure.

How well did you know this?
1
Not at all
2
3
4
5
Perfectly
26
Q

A 52-year-old woman with a history of diabetes and rheumatoid arthritis presents for her annual examination. She works in an office 10 hours a day, and rarely gets exercise. Her BMI is 23 and her blood pressure is 152/85. Her previous visit two months ago showed blood pressure of 148/82. Her father had a history of diabetes and her maternal grandmother died of rheumatic heart disease at the age of 42. She admits to marijuana drug use in the past and is a nonsmoker. Which of the following is a risk factor for coronary heart disease (CHD) that this patient has?

A. Age
B. Family history
C. Rheumatoid arthritis
D. Obesity
E. Lifestyle
A

The correct answer is E.
This patient’s lack of exercise and sedentary job are risk factors. Age becomes a risk factor over age 55 for women and 45 for men. The family history becomes a risk factor if a first degree relative has CHD male <55 and female <65. Rheumatoid arthritis is not a risk factor, and the patient is not obese.

How well did you know this?
1
Not at all
2
3
4
5
Perfectly
27
Q

Which of the following symptoms are most likely to be due to acute coronary syndrome?

A. 23-year-old male with acute onset of difficulty breathing and hyperresonance upon lung auscultation
B. 42-year-old woman with a pulsating pain in the center of her chest at night
C. 35-year-old man with chest pain radiating down his left arm after falling off a ladder at work one week ago
D. 59-year-old woman with palpitations that increase with exercise and are associated with nausea and vomiting
E. 55-year-old woman with diffuse central chest pain that is worse when lying down

A

The correct answer is D.
Palpitations, nausea, and vomiting are seen as prodromal symptoms of ACS in women more than men.
Young men with difficulty breathing and hyperresonance are likely to have a pneumothorax. Pulsating pain is one of the types of pain, along with pleuritic and positional pain, that is less likely to be related to heart disease. Chest pain in a young man associated with trauma is less likely to be ACS.

How well did you know this?
1
Not at all
2
3
4
5
Perfectly
28
Q

A 35 year old overweight woman in good health comes to the clinic for a routine physical. Which of the following screenings are recommended by the U.S. Preventative Services Task Force (USPSTF)?

A. Complete blood count
B. Thyroid stimulating hormone levels
C. Cholesterol panel
D. Blood pressure screening
E. HbA1C
A

The correct answer is D.
The only one of these that the USPSTF recommends in this age group is blood pressure screening. It is recommended to screen women with a cholesterol if there are risk factors for CHD.

How well did you know this?
1
Not at all
2
3
4
5
Perfectly
29
Q

Which of the following patients is an appropriate candidate for a exercise stress test?

A. A 58-year-old male who presents to the emergency room with constant substernal chest pressure, diaphoresis and shortness of breath
B. A 44-year-old female with a BMI of 40 kg/m2 and history of asthma
C. A 48-year-old female with a history of intermittent anginal episodes that have been controlled on medications.
D. A 52-year-old female with new atypical chest pain with a history of elevated cholesterol, smoking and family history of coronary artery disease

A

The correct answer is D.
An exercise stress test is useful if the pretest probability of the disease is high as in D with 3 significant risk factors. False positives increase if the pretest probability is low so the usefulness of a stress test is low in B. Likewise if the diagnosis is certain as in A then there is not as much use in getting a stress, especially in an unstable patient. Since the purpose of invasive treatment is symptom control, the patient in C has controlled symptoms and hence no need for stress testing.

How well did you know this?
1
Not at all
2
3
4
5
Perfectly
30
Q

A 62-year-old man with hypertension and diabetes is discharged home from the hospital, following a viral upper respiratory tract infection. He is prescribed several medications, including low dose aspirin. The patient asks you why he is taking aspirin along with the other medications. What is the reasoning behind your response?

A. Aspirin helps decrease the risk of ischemic stroke in men.
B. Aspirin helps decrease the risk of developing a hypercoagulable state.
C. Aspirin helps decrease the risk of myocardial infarction in men.
D. Aspirin decreases the risk of gastrointestinal hemorrhage.

A

The correct answer is C.
The USPSTF recommends initiating aspirin therapy in men age 45 to 79 years to reduce the risk of myocardial infarction. In women age 55 to 79, the USPSTF advises taking aspirin to reduce the risk of ischemic stroke. For both men and women, the benefit of decreased risk from those outcomes must be weighed against an increased threat of gastrointestinal hemorrhage. Other groups disagree that all men be treated, and only use aspirin for high risk patients, where the benefits outweigh the risk of GI hemorrhage, which is increased in patients taking aspirin.

How well did you know this?
1
Not at all
2
3
4
5
Perfectly
31
Q

Mr. Brown is a 42-year-old male accountant with a significant past medical history of obesity who presents to his primary care physician after one week of lower back pain. After moving into a new home three days ago, he woke up the next morning with bilateral lower back pain without any radiation. He denies any recent trauma, fever, chills, numbness, tingling, or incontinence. He has not had any urinary frequency or dysuria. He takes no medications and has no significant past medical history.
Which additional findings in his history or physical exam would make the diagnosis of lumbosacral sprain/strain more likely?

A. Increased pain with coughing
B. Abnormal gait
C. Point tenderness on spinous processes
D. Loss of ankle jerk
E. Spasm of paraspinous muscles
A

The correct answer is E.
Spasm of the paraspinous muscles suggests lumbosacral sprain/strain. Increased pain with coughing, abnormal gait and loss of ankle jerk point to conditions that compress a regional nerve root, while point tenderness on the spinous processes often indicates an origin in the vertebra (osteoporotic fracture, malignancy, etc.).

How well did you know this?
1
Not at all
2
3
4
5
Perfectly
32
Q

Mr. Giovanni is a 37-year-old male who drives a delivery truck. He presents to your clinic after acute onset of severe lower back pain, which began after lifting a large package while at work. When you enter the room, you find him standing, unable to sit comfortably.
On physical exam, he has limited lumbar flexion, reduced to 45 degrees, positive straight leg test at 45 degrees on the left, normal gait, but difficulty with heel walk. He has 4/5 strength on the left with ankle plantar flexion. Strength is preserved on the right.
Which of additional physical exam finding would be consistent with this man’s level of disc herniation?

A. Hypoactive ankle tendon reflex
B. Decreased range of motion on lumbar extension
C. 2/5 strength on hip flexion
D. Decreased rectal tone
E. Positive Stoop test
A

The correct answer is A.
The clinical signs presented by this patient-difficulty with heel walk and the abnormal strength of ankle plantar flexion-is consistent with nerve root impingement at the level of L5. Of the answers listed, a hypoactive ankle tendon reflex is also consistent with a nerve root impingement at this level. Pain with lumbar extension suggests degenerative disease or spinal stenosis, and spinal stenosis is similarly suggested by a positive stoop test. Diminished hip flexor strength suggests a lesion at the L2, L3, or L4 level and decreased rectal tone suggests a cauda equina lesion.

How well did you know this?
1
Not at all
2
3
4
5
Perfectly
33
Q

Mr. Roberts is a 78-year-old male with a significant past medical history of chronic kidney disease stage II, coronary artery disease, and hypertension who presents lumbar back pain. He has also been feeling general malaise and chills over the past few days. On review of symptoms he reports having some difficulty urinating with hesitancy and pain on urination.
Currently, his chronic conditions are well managed with metoprolol, lisinopril, and aspirin. Vital signs are temperature 100.2 F, blood pressure 135/75, pulse 76/min, and respiratory rate 15/min.
Given this history, which of the following physical exam maneuvers are most helpful in making the diagnosis? Select all that apply.

A. Abdominal palpation
B. Costovertebral angle percussion
C. Straight leg test
D. Digital rectal exam
E. Pinprick sensation of the legs
A

The correct answers are B and D.
In an older male patient, prostatitis may present with low back pain. This patient’s symptoms-general malaise, chills, hesitancy and pain on urination-and signs (fever) suggest acute bacterial prostatitis. Patients with acute bacterial prostatitis will often have exquisite tenderness over the prostate on rectal exam. This patient could also have pyelonephritis, which often goes along with costovertebral angle tenderness.

34
Q

Working at your clinic, you receive a call from a patient of yours, Mr Smith, a 45-year-old male who was seen three days ago complaining of lower back pain. At that time he had no history of trauma, pain that improved while lying down and no neurologic deficits. He works as a truck driver. He was treated conservatively along with pharmacologic intervention with NSAIDs and muscle relaxants.
He calls your office now due to only minimal improvement. And although his symptoms have not changed, he is frustrated with the slow progress, needs to get back to work as soon as possible, and is concerned this might be “something serious.”
Which of the following is the most appropriate next step in management?

A. Obtain a plain film x-ray
B. Order an MRI
C. Ask him to double the dosage of his muscle relaxants
D. Schedule him for an appointment immediately
E. Reassure him and schedule a follow-up appointment in a few days

A

The correct answer is E.
Given this clinical presentation, the likelihood of this being an episode of lumbar sprain/strain is high, and the odds of this being “something serious” (nerve root compression, malignancy, infection) is still low. The original plan is a good one, and should continue. No new meds or imaging studies would help, and an urgent appointment will not change the anticipated course. Some physicians might choose to involve a physical therapist at this time, but this option is not available for this question.

35
Q

Ms. Vasquez is a 38-year-old female with a past medical history of sarcoidosis and recently completed a six-month steroid taper. She presents to her primary care physician after two weeks of lower lumbar back pain. She does not recall any trauma but began to feel a sharp pain after bending over to pick up laundry. The pain radiates bilaterally into her anterior abdomen. She has found no relief with over the counter NSAIDs. On physical exam, she has point tenderness along her vertebrae in the L1-L2 region. There are no neurologic deficits and reflexes are intact.
Which of the following is the most appropriate next step in management?

A. Reassess in four weeks
B. Refer to spine specialist
C. Order complete blood count
D. Order a plain x-ray
E. Recommend conservative management
A

The correct answer is D.
The “red flag” in this history is that of chronic steroid use, and the concomitant risk of osteoporotic vertebral fracture. A vertebral fracture is best diagnosed with a plain x-ray. A CBC will not help with the diagnosis, and referral to a spine specialist is unnecessary. Conservative management and/or reassessment in four weeks demonstrate a failure to recognize the “red flag.”

36
Q

You are seeing one of your regular patients, a 65-year-old female for a follow-up appointment for intractable knee pain from osteoarthritis. The knee pain has not responded to ibuprofen or acetaminophen. She has a past medical history that also includes obesity, diabetes, hypertension and depression. Her current medications include aspirin, HCTZ, metformin, and duloxetine. You are considering prescribing tramadol. She has never taken any kind of opioid medication in the past.
Which of the following potential problems should you counsel the patient about when adding tramadol?

A. Hypotension
B. Seizures
C. Rash
D. Agitation
E. Hypercoagulability
A

The correct answer is B.
Tramadol is a centrally-acting synthetic opioid analgesic that works by binding to mu-opioid receptors and weakly inhibiting norepinephrine and serotonin reuptake. Significant side effects can include seizures, serotonin syndrome, respiratory depression, angioedema, bronchospasm and dependency. Other common side effects include constipation, nausea, dizziness, and pruritis.

37
Q

A 59-year-old patient comes to the local ER with a swollen, tender knee that started yesterday. He returned home two days ago from a 5-day hike on the Appalachian Trail. He denies any recent or previous injury to the knee and any history of previous inflammatory joint disease. Vital signs: temperature is 97.7, pulse is 80, BP is 139/75, and RR is 22. He holds the knee in full extension. The knee is swollen, reddened, tender, and it feels warm. Synovial joint fluid aspiration is done.
Which of the following bedside observations of the aspirate would be inconsistent with the presentation?

A. Yellow-green fluid
B. Cloudy fluid
C. Pink or red fluid
D. Straw-colored fluid

A

The correct answer is D.
Straw-colored fluid is the color of normal synovial fluid, and would not be an expected finding in this patient with probable septic arthritis. Yellow-green fluid indicates an increased number of plasma/nucleated cells, seen in inflammatory and septic arthritis. Cloudy fluid has an increased opacity due to an increased number of WBC’s or crystals. Pink or red fluid indicates blood, and can be present if there was a traumatic tap done when collecting the sample.

38
Q

Preliminary results from the joint aspirate for the prior patient above show the following:
Cell count: >100,000/mm3
Gram stain: positive
Crystals: negative
Which of the following would be the next appropriate step in diagnosis?

A. ESR and CRP
B. Blood culture
C. Immediate surgical exploration
D. Urinalysis
E. Synovial fluid PCR
F. Synovial fluid culture
G. Steroid injections into joint space
A

The correct answer is E.
Synovial fluid PCR can aid in the diagnosis of arthritis caused by Yersina, Chlamydia, Ureoplasma urealyticum, N. gonorrhoeae, and in this case, Borrelia burgdorferi. PCR has a > 90% sensitivity for Lyme disease, and helps confirm the diagnosis. Synovial fluid culture is insensitive in extracutaneous manifestations of Lyme. ESR and CRP are elevated in inflammatory processes, but are not reliably elevated in septic arthritis. This patient does not appear septic based upon his physical exam, making blood cultures less urgent. Immediate surgical exploration is overly aggressive as the next step, urinalysis would not help in the diagnosis of this patient, and septic arthritis can complicate joint space injections and should not be the next step for this patient’s work-up.

39
Q

A 51-year-old woman comes to you with acute pain and swelling of the knee. Joint fluid analysis confirms the diagnosis of acute gout. Which of the following information from her history would dissuade you from initiating NSAID therapy?

A. Her age
B. Currently on warfarin
C. Previous H. pylori infection
D. Current UTI
E. Hypothyroidism
A

The correct answer is B.
NSAIDs can increase the effect of anticoagulants, and should be avoided in patients taking warfarin. NSAIDs do increase the risk of GI bleeding and peptic ulcer disease in the elderly (older than 75), but this patient is only 51-years-old. The other answers would not contraindicate NSAID use.

40
Q

On the prior patient you determine NSAIDs are not an appropriate option. In considering how to treat this woman, what is the next question you should ask?

A. Who in her family also has gout?
B. Has she recently traveled outside the country?
C. Are there any other joints involved besides her knee?
D. Has she ever received steroid injections in the past?

A

The correct answer is C.
In patients with contraindications to NSAIDs or colchicine, the choice of treatment can be guided by the number of joints involved. If one or two joints are involved, arthrocentesis with intra-articular glucocorticoid injection is appropriate. For polyarticular disease, oral glucocorticoids would be a more appropriate option.

41
Q

A 35-year-old man presents to clinic with acute onset of constant right upper quadrant abdominal pain. Additionally, he complains of ongoing nausea and vomiting. He denies any past abdominal surgeries or chronic diseases and his only medication is a multivitamin. His AUDIT-10 was positive. What lab(s) would you most likely expect to be abnormal to confirm your diagnosis of acute alcoholic hepatitis?

A. GFR and INR
B. CRP and alkaline phosphatase
C. AST, ALT and total bilirubin
D. AST, ALT, INR and WBC
E. Lipase and amylase
A

The correct answer is C.
It is expected that someone with acute alcoholic hepatitis would have an elevated AST, ALT and total bilirubin. GFR (glomerular filtration rate) is a marker of kidney function and is often not abnormal in early hepatitis. INR is a marker of synthetic functioning of the liver and would be elevated in end-stage liver disease. WBC is often elevated in cholecystitis and infectious processes. Elevation of lipase and amylase is associated with pancreatitis.

42
Q

A 35-year-old man presents to clinic with acute onset of abdominal pain. On abdominal exam, you flex the patient’s right hip to 90 degrees and take his right ankle in your right hand and with your left hand externally then internally rotate his hip by moving the knee back and forth. The patient denies any abdominal pain with this movement. What physical exam sign did you just perform and what disease is it ruling out?

A. Psoas sign to rule out appendicitis
B. Psoas sign to rule out cholecystitis
C. Obturator sign to rule out appendicitis
D. Murphy’s sign to rule out appendicitis
E. Obturator sign to rule out cholecystitis

A

The correct answer is C.
During an abdominal exam, it is important to rule out other causes of abdominal pain. Appendicitis is another cause of acute abdominal pain and can be ruled out with a number of signs. One of these signs is the Obturator sign and the technique is discussed in the question stem. Another sign to rule out appendicitis is the psoas sign. This sign includes passive extension of patient’s thigh as they lie on their side with their knees extend, or asking the patient to actively flex their thigh and hip. Pain with movement is often indicative of appendicitis due to inflammation and irritation of the psoas muscle. Murphy’s sign is to identify cholecystitis. The technique involves having the patent breathe out slowly and completely, then gently placing your hands under the right costal margin. The patient is then instructed to take a deep breath in while you palpate for a hardened mass and determine if you elicit any significant tenderness.

43
Q
A 42-year-old woman presents with nausea, vomiting and RUQ pain radiating to her back for one day. She reports a history of similar episodes but none have ever lasted for this long of a time period. Her vital signs are HR of 108, BP of 145/90, RR of 20, O2 saturation of 98% and temperature of 100.8F. Her labs reveal the following:
WBC = 14.0 x 103/mL
AST = 55 U/L
ALT = 60 U/L
Amylase = 70 U/L
Lipase = 7 U/L
What is the most likely diagnosis?
A. Biliary colic
B. Acute cholecystitis
C. Acute pancreatitis
D. Duodenal ulcer
E. Hepatitis
A

The correct answer is B.
Acute cholecystitis has similar pathophysiology to biliary colic and often results from a bile stone that is lodged in the biliary tree. However, there are additional pathophysiologic reasons for acute cholecystitis. The symptoms are similar to biliary colic but typically last longer than 4-6 hours and may be more severe. Symptoms may include fever and elevated WBC. Biliary colic typically lasts 4-6 hours or less, radiates under right shoulder blade, often is accompanied by nausea, vomiting and can often follow a heavy, fatty meal. The hallmark of biliary colic is the stone is still mobile and gallbladder function resumes with relief of symptoms. Acute pancreatitis is often difficult to distinguish from biliary colic but will have an increased lipase and amylase. A patient with duodenal ulcer typically has epigastric pain that is relieved by food or antacids. Hepatitis is usually distinguished by malaise, anorexia, itching and icterus or jaundice. Signs of hepatitis would include hepatomegaly and elevated transaminases.

44
Q

A 24-year-old man presents to clinic with history of upper abdominal pain, nausea and vomiting. He has previously had his gallbladder removed due to symptomatic gallstones. In reviewing his history, you want to screen for alcohol abuse due to the possibility of pancreatitis. Which one of the following meets criteria for alcohol abuse:

A. Failure to fulfill work, school or social obligations
B. Denial of a drinking problem
C. Score of 4 or more on AUDIT-C
D. 10 drinks per week
E. 3 drinks per social occasion
A

The correct answer is A.
Alcohol abuse is characterized by a maladaptive pattern of alcohol use with one or more of the following: failure to fulfill work, school or social obligations, recurrent substance use in physically hazardous situations, recurrent legal problems related to substance use, or continued use despite alcohol-related social or interpersonal problems. Further screening for alcohol abuse and/or dependence can be done with the AUDIT-10.

45
Q

A 58-year-old man presents to clinic with right upper quadrant pain. The patient has a history of hypertension and GERD, but denies any past surgeries. His abdominal exam reveals a positive Murphy’s sign. What would be the next best step to confirm your working diagnosis?

A. Complete blood count
B. Liver function tests
C. CT abdomen without contrast
D. CT abdomen with contrast
E. Abdominal ultrasound
A

The correct answer is E.
Real-time ultrasonography is the preferred study to evaluate right upper abdominal quadrant because it is inexpensive, noninvasive and widely available. US provides good evaluation of gallbladder and is accurate in detection of gallstones as well as dilation of biliary tree. A CT with contrast may give you some evidence of gallbladder disease but is not the preferred test when suspecting biliary colic or cholecystits. A CT without contrast is used primarily when there is a suspicion for kidney stones. The other tests would help with your diagnosis, but US is preferred method for biliary colic/gallstones.

46
Q

A 47-year-old caucasian female presents to the clinic complaining of an “itchy patch” on her skin. On further examination, you note a solid, elevated 1.5cm lesion on the extensor surface of the right forearm. How would you best describe the lesion?

A. Plaque
B. Papule
C. Patch
D. Nodule
E. Macule
F. Bulla
G. Pustule
A

The correct answer is A.
A plaque is a palpable flat lesion that is greater than 1cm in size. Papules are small raised palpable lesions that are less than 1cm in size. Nodules are solid raised palpable lesions which are larger in height and dimension than papules, occurring in the epidermis, subcutaneous tissue, or dermis. A macule is an area of skin discoloration that is 0.5-1cm in diameter, and a patch is a larger area of differently colored skin which is smooth to touch. A pustule is a small pus-filled raised lesion, while a bulla is a larger (greater than 1cm) fluid-filled raised lesion.

47
Q

A 12-year-old boy is brought to the clinic with a linear maculopapular rash extending down both of his arms. He has just returned from a summer camp program in Wisconsin. Excoriation marks are visible. Which of the following is the most appropriate course of action?

A. Biopsy the skin lesion
B. Ask the patient to return to the clinic after 3 days
C. Prescribe topical corticosteroids
D. Prescribe oral antibiotics
E. Prescribe oral corticosteroids
A

The correct answer is C.
This child appears to have poison ivy. People affected by poison ivy tend to have linear lesions, due to contact with a branch or leaf or due to self-inoculation by scratching. The lesion are often fluid-filled, or vesicular, but over time can have less vesicular appearance. Treatment of choice is corticosteroids. For milder conditions, such as the case here, topical steroids are effective. For more severe cases in which a larger portion of the body is affected, oral steroids may be indicated. Antibiotics are not necessary, since this is not due to an infection. Biopsy is not necessary due to typical appearance and history. Treatment is important in order to reduce symptoms, so waiting 3 days would not be indicated.

48
Q

A 26-year-old professional football player comes to the clinic with the complaint of hair loss. On examination, the scalp is scaly, erythematous, and certain regions are purulent. There are several circular spots where the hair follicles are no longer present. KOH of skin shows hyphae. What is the most appropriate treatment for this patient?

A. Punch biopsy of lesion
B. Topical griseofulvin
C. Oral griseofulvin
D. Oral prednisone
E. Topical prednisone
A

The correct answer is C.
The description given in this scenario is suggestive of tinea capitis. Tinea capitis requires oral antifungals, rather than topical treatments. Topical therapies do not penetrate the infected hair shaft. Prednisone is not the treatment of choice for this condition. Biopsy is not necessary due to the typical appearance of this condition.

49
Q

A 64-year-old African American man with a history of frontotemporal dementia is brought to the doctor for multiple dark spots found on the palms of his hands. The lesions are asymmetric and some have more than one color. Based on the information provided, which is the most likely finding on the biopsy?

A. Nodular melanoma
B. Acral lentiginous melanoma
C. Squamous cell carcinoma
D. Superficial spreading melanoma
E. Benign nevus
A

The correct answer is B.
Acral lentiginous melanoma is seen more often in dark-skinned people, and typically appears on the palms and soles of feet, including under the nails. Nodular melanoma presents as a single dark brown or black nevus on a sun-exposed area that grows deep into the skin. Superficial spreading melanoma presents as a nevus that has been growing and spreading along the skin surface. Benign nevi appear as small, symmetric, uniform colored moles. Squamous cell carcinoma tends to have a scaly, erythematous appearance more typical of a patch, plaque or nodule.

50
Q

A 57-year-old Caucasian woman comes in, concerned about a 1.5 cm dark multicolored mole on her chin that has been increasing in size over the past 6 months. An excisional biopsy shows pathology indicative of squamous cell carcinoma with clear margin. What is a preventative measure that the patient could implement to prevent further recurrences?

A. Use of sunscreen with an SPF of at least 15
B. Avoid artificial sources of UV light, such as indoor tanning
C. Visit the doctor every six months for a whole body skin examination
D. Wear a wide brimmed hat when in direct sunlight
E. All of the above

A

The correct answer is E.
All of the above are important measures to take for skin cancer prevention in someone who has already been diagnosed with one cancerous lesion. All of these measures are appropriate preventative measures for those who have no skin cancer as well, except the frequency of screening exams would be annually instead.

51
Q

A 53-year-old woman with a past medical history of diet-controlled hypertension presents to the office with a two-month history of worsening hot flashes. Her menstrual cycles are regular, occurring every 30-32 days, but they have gradually lessened in duration, now lasting 4-5 days instead of the previous 6-7 days. Vital signs and physical exam are normal. Which one of the following treatments is most likely to improve the patient’s symptoms?

A. Black cohosh
B. Oral estrogen
C. Venlafaxine
D. Gabapentin
E. Yoga
A

The correct answer is B.
While all of the listed items have been touted for treatment of hot flashes, estrogen is clearly the most effective. There is evidence that venlafaxine and gabapentin help some. There is no convincing evidence that black cohosh or other herbal medications help, as there is no evidence that yoga, mindfulness or exercise help.

52
Q

A 56-year-old woman with a past medical history significant for hypothyroidism and recurrent urinary tract infections presents to a local health fair to discuss dyspareunia and hot flashes that began approximately one year ago. Her last menstrual cycle was 14 months ago, and her family history is significant for breast cancer in her sister, diagnosed at the age of 47. She has become increasingly bothered by the frequency of her hot flashes and has stopped attending social events with her friends due to these symptoms. Which one of the following is true regarding the risks and benefits of hormone-replacement therapy (HRT)?

A. Use of combined estrogen and progesterone therapy decreases the risk of breast cancer.
B. Use of unopposed estrogen in patients with an intact uterus decreases the risk of endometrial cancer.
C. Use of hormone therapy for less than five years will not affect a patients risk of coronary artery disease.
D. Use of hormone therapy decreases the risk of osteoporotic fractures.
E. Both systemic and topical estrogens are equally effective for the treatment of vasomotor symptoms.

A

The correct answer is D.
Use of combined estrogen and progesterone beyond three years increases the risk of breast cancer. Use of unopposed systemic estrogen in women with an intact uterus increases endometrial cancer risk. Beginning HRT after age 60 increases the risk of coronary artery disease. Systemic estrogen is most effective for treatment of vasomotor symptoms. HRT has been shown to decrease the risk of osteoporotic fractures.

53
Q

A 52 year-old female with no past medical history presents to your office with amenorrhea. The patient states that her menstrual cycles previously occurred approximately every 28-34 days. However, she has not had a menstrual cycle for the last 10 months. She also endorses insomnia and intermittent dysuria. She denies any headaches, abdominal pain, constipation or diarrhea, changes in hair distribution, or easy bruising. She has lost 15 pounds since her last visit eight months ago, which she attributes to improving her diet and beginning regular exercise. Which one of the following tests can be used to confirm your diagnosis?

A. Thyroid Stimulating Hormone (TSH) and Free T4
B. Luteinizing Hormone (LH) and Follicular Stimulating Hormone (FSH)
C. Prothrombin Time (PT) and International Normalized Ratio (INR)
D. Morning Cortisol and Prolactin
E. Testosterone and Dehydroepiandrosterone sulfate (DHEA-S)

A

The correct answer is B.
Elevated FSH and LH levels can be used to confirm menopause. During menopause, ovarian granulosa cells produce less inhibin, thereby affecting the negative feedback loop of FSH and LH secretion from the pituitary gland. TSH and free T4 are used in testing thyroid function. One might be concerned about thyroid function if there was unexplained weight loss along with other positive symptoms in the ROS, but those are not present in this case. PT and INR are used to test bleeding time and can be an indicator of liver function, while morning cortisol tests adrenal function - and neither are of concern in this patient. Prolactin is produced by the pituitary but is not affected during menopause.

54
Q

A 61 year-old G4P4 female presents to a local emergency room with vaginal bleeding. The bleeding began two days ago and is described as spotting. She began her menses at age 16 and had regular menstrual cycles until the age of 59. She endorses smoking ½ pack per day for the last 23 years and drinks 1-2 glasses of wine with dinner every evening. She denies any abdominal pain or dysuria. She is 5’7” and weighs 112 lbs. You perform a physical exam, including a vaginal exam, pap smear, and bimanual exam. Which one of the following characteristics of this particular patient increases her risk of endometrial cancer?

A. Smoking
B. Multiparity
C. Body habitus
D. Age of menarche
E. Age of menopause
A

The correct answer is E.
Any characteristic that increases exposure to unopposed estrogen increases the risk of endometrial cancer. This includes Tamoxifen therapy, obesity, anovulatory cycles no prior history of pregnancy, early menarche (before age 12), and late menopause (after age 52). Smoking decreases estrogen exposure, thereby decreasing risk. Oral contraceptives increase progestin levels, thus providing protection against endometrial cancer.

55
Q

An otherwise healthy 57 year-old G2P2 female presents to your office with vaginal bleeding that began one week ago. She began her menses at age 13 and had regular menstrual cycles until the age of 49. She denies any tobacco or alcohol use. Further review of systems is negative. You perform a physical exam, including a vaginal exam, pap smear, and bimanual exam. Which of the following is the most appropriate next step?

A. Reassurance
B. Trial of oral contraceptives
C. Transvaginal ultrasound
D. Endometrial ablation
E. Referral for hysterectomy
A

The correct answer is C.
Any postmenopausal bleeding needs to be investigated; reassurance alone is inappropriate in this case. A transvaginal ultrasound is the most cost-effective initial test in women with abnormal uterine bleeding at low-risk for endometrial cancer. It is highly sensitive (96%) for the detection of endometrial cancer. Endometrial biopsy is considered the gold standard for evaluation of post-menopausal bleeding and is up to 99% sensitive. Oral contraceptives, endometrial ablation, and hysterectomy are treatment modalities that would not be appropriate until an actual diagnosis is made.

56
Q

A 34-year-old male comes to the clinic complaining of abdominal pain. He says the pain has been bothering him for the past two weeks. He reports episodes of diarrhea and constipation, with more episodes of constipation. He states he has noticed an increase in flatulence. He denies any nausea or vomiting. He has noticed mucus in his stools, but no blood. He states that he cannot recall if anything aggravates the pain, but admits to being under more stress than usual, due to his mother-in-law moving in with him and his wife. Vital signs show a blood pressure of 124/76 mmHg, pulse of 74, respirations of 16, a temperature of 97.9, and oxygen saturation of 98% on room air. Physical exam is unremarkable.
The most appropriate initial step in management is:

A. Scheduling the patient for a colonoscopy to look for colon cancer
B. Discussing the patients diet, and educating him about avoiding dairy products
C. Performing a CBC, TSH, complete metabolic panel, and stool studies
D. Prescribing an antispasmodic
E. Scheduling the patient for a CT scan of the abdomen to rule out small bowel obstruction

A

The correct answer is C.
The patient most likely has a diagnosis of irritable bowel syndrome. When a patient presents with abdominal pain, mucus in stool, episodes of diarrhea and/or constipation, it is appropriate to perform lab studies to rule out other etiologies, such as infection, autoimmune disease, or an obstruction. Since IBS is a diagnosis of exclusion, the diagnosis can be made once other sources of abdominal pain have been ruled out by appropriate studies.
Scheduling a colonoscopy to search for colon cancer is inappropriate, as he is not presenting with bloody stool or significant weight loss. The patient is too young to undergo a screening colonoscopy without a history of a first degree relative who was diagnosed with colon cancer at an age 10 years older than his current age.
Discussing the patient’s diet is suitable, a trial of avoidance of dairy products would be dependent on history and a recommendation that includes this strategy may or may not be appropriate.
Prescribing an antispasmodic would be appropriate after a diagnosis of IBS is made. Antispasmodics, such as Bentyl or Anaspaz, are pharmacological agents used to treat symptoms, and are not usually used long-term.
Scheduling a CT scan of the abdomen to rule out a small bowel obstruction is inappropriate. The patient is not complaining of nausea or vomiting, and has no abdominal tenderness or hyperactive bowel sounds on physical exam, all of which would be suspicious for small bowel obstruction.

57
Q

A 24-year-old female presents to the clinic complaining of a headache and arm pain. She is accompanied by her boyfriend, who insists on staying in the room with her during the visit. When asked to describe when she noticed the pain and how it occurred, the boyfriend states that the patient hit her head on the bathroom cabinet two days ago, and fell on her side afterwards, hitting her arm. The patient nods in agreement, while looking down at the floor.
The next best step in obtaining an accurate history and physical exam is to:
A. Direct all questions to the boyfriend, since it appears the patient does not want to discuss her situation with you
B. Accuse the boyfriend of domestic abuse and call the police to report him
C. Ask the boyfriend to leave the room so you can perform the physical exam, and explain that you always do that part of the exam with just the patient, and that he may join you after that part is completed
D. Give the patient referrals to neurology and orthopedics to evaluate the patients headache and arm pain
E. Prescribe Tylenol 650mg po BID to manage the pain, and tell the patient to return in a week if symptoms have not disappeared

A

The correct answer is C.
When domestic violence is suspected and the partner is present for the visit, the best step in obtaining an accurate history and physical from the patient is to politely but firmly ask the partner to leave the room. To aid you in asking the partner to leave, it is suitable to tell them that the next part of the exam is done with the patient independently, and the partner may rejoin you and the patient after the exam is performed. ** If the partner still insists on staying, use other tactics, such as a test performed outside of the exam room or by taking the patient to get a urine sample.
Directing questions to the boyfriend instead of the patient is inappropriate. You should always direct all questions to the patient.
Accusing the boyfriend of domestic abuse and calling the police could precipitate the boyfriend acting out in the office towards you or the patient. It could also damage the physician-patient relationship, since it is not proven that the boyfriend is engaging in domestic violence towards the patient.
Referring the patient to neurology and orthopedics is inappropriate. Initial workup for headache and arm pain can be done in the clinic. Referrals would be appropriate after the exam and workup are done, and not as an initial step to obtain a history and physical.
Prescribing Tylenol would not be a step towards gathering a history and physical for the patient, and would be done at the end of the office visit.

58
Q

A 22-year-old female comes to the clinic complaining of frequent urination. She states she has noticed some increased frequency for the past week. She denies itching or pain in the vaginal area, but reports a burning sensation during urination that began a week ago. When asked if she has noticed blood in her urine, she admits to not paying attention to its color. She states that she has some abdominal pain in her pelvic area. She admits to being inconsistent with her birth control pills, and states she is sexually active. She does not recall the date of her last menstrual period. Urine hCG test is negative. Physical exam reveals suprapubic tenderness. She exhibits no costovertebral angle tenderness. Vital signs show a blood pressure of 126/78 mmHg, temperature of 98.2, respirations of 18, heart rate of 82, and oxygen saturation of 97%.
The most likely diagnosis is:

A. Pyelonephritis
B. Urosepsis
C. Pregnancy
D. Uncomplicated urinary tract infection
E. Vaginitis
A

The correct answer is D.
A urinary tract infection presents with increased urinary frequency, dysuria, urinary urgency, and suprapubic pain. It can be associated with burning upon urination. Risk factors for UTIs include sexual intercourse, female gender, pregnancy, the presence of an indwelling urinary catheter, structural abnormalities (BPH, neurogenic bladder,etc.), immunocompromised states, diabetes, spinal cord injuries, and a history of recurrent UTIs. **
Pyelonephritis would present with fever, chills, and flank pain. It often is associated with urinary symptoms, such as increased frequency and dysuria. Nausea and vomiting may be present. Physical exam for a patient with pyelonephritis would reveal tachycardia, fever, and positive costovertebral angle tenderness. They may exhibit abdominal tenderness. The patient does not have fever, CVA tenderness, and is not complaining of symptoms that are indicative of pyelonephritis.
Urosepsis would present with acute onset of fever, chills, tachycardia, tachypnea, and altered mental status. This patient does not have any of these presenting signs.
Pregnancy does involve increased urinary frequency, and it a risk factor for UTIs. However, it is not a likely diagnosis in this case, as the patient’s urine hCG is negative.
Vaginitis would present with vaginal itching or irritation, as well as vaginal discharge. Examination of the discharge is necessary to discover the etiology. This patient is not complaining of any vaginal discharge or itching, making this a less likely diagnosis.

59
Q

A 32-year-old female comes to the ER complaining of abdominal pain. She states the pain has been bothering her for the last week, and has progressively become worse. She denies nausea, vomiting, any episodes of diarrhea, or chest pain. She states she was diagnosed with high blood pressure six months ago, and is currently taking medication for it. She is sexually active with her boyfriend, and admits to having some pain during sex. She states she does not feel comfortable telling her boyfriend that she does not want to have sex. Her last menstrual period was two weeks ago. She denies a history of sexually transmitted infections, but says that she has not questioned her significant other because “he would be upset.” Vital signs reveal a temperature of 98.7, blood pressure of 142/90 mmHg, heart rate of 82, and respirations of 18. The patient is visibly uncomfortable during the physical exam, which shows a few bruises on her abdomen and lower back, which are at different stages of healing.
Which of the following is an appropriate technique for screening for domestic violence?

A. Request an appointment with the patients significant other, and asking him about abusing the patient
B. Ask if she had ever been sexually abused as a child
C. Ask the patient, “all couples disagree at some point in time. What happens when you and your partner argue or disagree?”
D. Stand above the the patient while asking them questions
E. Avoid direct questions about guns in the house or about drug/alcohol use by them or their partner

A

The correct answer is C.
To screen for domestic violence in a patient should involve asking indirect questions that include the opportunity to directly assess the safety of the the patient and their environment.
Asking the patient’s significant other to be present when screening for domestic violence could alter the answers of the victim, as well as damage the doctor-patient relationship.
Sexual abuse in the past whether present or not, doesn’t preclude the immediate issue of potential partner abuse/violence.
Standing above the patient during the screening interview can intimidate the patient.
Sitting down at the level of the patient allows for the intimidation to be less of a factor. It is appropriate to ask about drug and alcohol use, as well as the presence of guns in the victim’s home environment in order to assess safety.

60
Q

A 23-year-old female comes to the clinic to discuss infertility. She states she was treated for an STD two years ago, and remembers receiving a Rocephin shot and taking a course of doxycycline. She admits to being hospitalized last year due to a severe infection with gonorrhea. She states that she and her fiance have attempted to have a baby for the past year, without success. She reports no dysmennorhea and has regular menses, but has pain during intercourse. Her last menstrual period was 4 weeks ago. Urine hCG is negative. Vital signs stable. Physical exam is unremarkable.
The most likely cause of the patient’s inability to conceive is:

A. Pelvic inflammatory disease
B. Uterine leiomyoma
C. Endometriosis
D. Candidiasis
E. Ovarian cyst
A

The correct answer is A.
The patient most likely was treated for pelvic inflammatory disease twice in the past. The most common cause of PID is Neisseria gonorrhoeae and/or Chlamydia trachomatis. It is treated with ceftriaxone and doxycycline to cover both bacteria. Pelvic inflammatory disease can cause infertility due to scarring of the fallopian tubes/ tubal adhesions.
Uterine leiomyomas are benign neoplasms of the female genital tract, and are usually asymptomatic. They are not treated with antibiotics. They are unlikely to cause infertility, unless they are submucosal and distort the endometrial cavity of the uterus in a way that makes it an undesirable environment for pregnancy.
Endometriosis involves functional endometrial tissue outside of the uterus, and often causes cyclical pelvic pain. Less commonly it may cause painful intercourse. Treatment may include surgery or a course of oral contraceptive pills to stop ovulation, or NSAIDs to treat symptoms. It can cause infertility, however is not the most likely cause of infertility in this patient due to her history.
Candidiasis is caused by an overgrowth of yeast that is found naturally in a female’s genital tract. The treatment is fluconazole. Candidiasis does not fit this patient’s history, and is not a common cause of infertility.
Ovarian cysts present with lower abdominal and pelvic pain. It may be severe pain if the cyst has ruptured. They are not treated with antibiotics making this less likely to fit the patient’s history or be the cause of this patient’s infertility.

61
Q

You are working in a free clinic for patients who are uninsured and mostly from a very low socioeconomic class. You’re seeing a 45-year-old patient whose only complaint is intermittent headaches and a family history significant for a brother who recently died at the age of 47 due to colon cancer. The patient states that he is concerned about developing colon cancer like his brother and wants to know if he should be screened; however, he cannot afford a colonoscopy. Your county has a government-funded colonoscopy screening program for patients who are asymptomatic and 50 years old.
For the time being, what testing could you recommend to your patient based on his means as a secondary screening method for colon cancer?

A. Yearly abdominal x-rays
B. Yearly fecal occult blood testing
C. Yearly hemoglobin checks
D. Yearly sigmoidoscopy

A

The correct answer is B.
While colonoscopy is still the recommended screening test for colon cancer and screening in first degree relatives is recommended to start 10 years before age of diagnosis and you will advocate for your patient to have this covered because of his family history, annual home based sensitive FOBT (fecal occult blood testing) has the same life years gained as colonoscopy every 10 years and is more affordable for those uninsured.
Abdominal x-rays are not effective in screening for colon cancer.
Yearly hemoglobin tests would identify if the patient is losing a significant amount of blood due to colorectal cancer, but anemia can be a later stage sign.
Sigmoidoscopy is a reasonable alternative, but is recommended every 5 years in conjunction with FOBT.

62
Q

You are seeing a 55-year-old established male patient of yours in the office for his yearly physical. He has a past medical history significant for hypertension and hyperlipidemia with no significant family history. He has no complaints and your review of systems and physical exam are all within normal limits. However, he would like you to write him an order for a PSA test because a friend of his recently passed away from prostate cancer.
What would be the best response to this patient’s request concerning checking PSA and prostate cancer?

A. Many men are understandably concerned about prostate cancer. A negative test can be very reassuring.
B. Studies have shown significant survival advantages to checking PSA levels in men with proper follow-up testing.
C. I can see that you are concerned about having prostate cancer. PSA screening in asymptomatic men is not recommended, but I would be happy to discuss this with you.
D. PSA is best used to track the progression of prostate cancer and getting a baseline now when you dont have symptoms can be advantageous.

A

The correct answer is C.
The United States Preventative Services Task Force recommends against PSA screening. It is still recommended that you discuss their concerns and the risks and benefits of PSA testing with patients.
A is wrong because while a negative test can be reassuring, the anxiety and testing caused by a positive test in most cases is worse and unnecessary.
B is not the best choice because early detection does not result in improved outcomes.
C is incorrect because while it is true that PSA is helpful in tracking progression of disease, getting a baseline is not felt to have significant benefit.

63
Q

You are seeing a 55-year-old female patient for follow-up who you met two weeks ago at her physical exam. Her review of systems and physical exam were only significant for fatigue. Her blood work shows iron deficiency anemia. She is post-menopausal x 2 years with no significant past medical history or family history but a 10 pack per year smoking history. Prior to establishing with you she hadn’t seen a doctor since she was 18 years old.
Which of the following would be the most appropriate next step in diagnosis?

A. Chest x-ray
B. Colonoscopy
C. Hemoglobin electrophoresis
D. Pelvic ultrasound

A

The correct answer is B.
Colonoscopy is recommended at age 50 and blood loss in a non-menstruating female is commonly GI related.
Chest x-ray in absence of other respiratory symptoms would not be indicated.
Hemoglobin electrophoresis would be indicated when hemoglobinopathies are suspected and not for iron deficiency which suggests blood loss.
A pelvic ultrasound in absence GYN symptoms in a post-menopausal woman would not be indicated.

64
Q

Ms. Wicket is a 30-year-old woman establishing with you as a new patient. Her past medical history is significant for hypothyroidism for which she has been without treatment for five years. She currently takes no medication and has no surgical or family history. She is single, with no children, drinks alcohol socially, and does not use tobacco or other recreational drugs. The only records she has are from blood work that she had done last month significant for a very elevated thyroid stimulating hormone (TSH).
What constellation of symptoms would you expect to find on your review of systems?

A. Dry skin, sensitivity to cold, constipation, and fatigue
B. Palpitations, sweating,lightheadedness, and fatigue
C. Anxiety, frequent bowel movements, sweating and increased appetite
D. Tremor, hair changes, difficulty sleeping and irregular menses

A

The correct answer is A.
Goiter and weight gain are also common symptoms.
Answers B, C, and D would be more consistent with hyperthyroidism.

65
Q

You are seeing a 42-year-old established female patient of yours in the office for her yearly physical. She has a past medical history of hypertension only, with no significant family history. She is a tenured professor at the local university. She has no complaints and your review of systems and physical exam are all within normal limits. However, she would like you to write her an order for a colonoscopy because a friend of hers was just diagnosed with colon cancer and passed away. You explain that based on USPSTF recommendation she doesn’t need to start colon cancer screening until she is 50 years old.
What would not be an appropriate statement to help support these guidelines and educate your patient?

A. All colonoscopy procedures have risks that we try to minimize by screening only when necessary.
B. Early screening is usually only recommended in patients with a first-degree relative or personal history of colon cancer.
C. Most cancerous colonic polyps don’t start growing until patients are about 50 years old and usually take about 10 years to mature.
D. While a screening colonoscopy is not indicated here, a sigmoidoscopy is recommended.

A

The correct answer is D.
In asymptomatic patients with a negative family history, no colon cancer screening is recommended before the age of 50.
The remainder of these answers are accurate and support the current guidelines.

66
Q

Ms. Martinez, 74-years-old, is brought to your family practice by her husband. He is concerned because his wife, who used to take pride in keeping the house in good shape, has stopped cleaning and caring for her appearance. She recently went shopping and left the stove on. She frequently misplaces her car keys and checkbook. Ms. Martinez reports trouble concentrating and does not have much energy. She has a poor appetite and no longer enjoys knitting. Her mini-mental status exam is 28/30. Physical Exam: General Appearance: well-nourished, appears stated age, HEENT[LW1]: pupils equally round and reactive to light, moist mucus membranes, Chest: normal S1 and S2, no murmurs, Lungs: clear to auscultation bilaterally, Abdomen: soft, non-tender, non-distended, Extremities: no cyanosis, no clubbing. Neurologic: no focal deficits Psychiatric: flat affect. Labs: TSH: 2, WBC: 6.0, HgbA1c: 4.5.
Which of the following is the most likely diagnosis?

A. Normal bereavement
B. Dementia
C. Depression
D. Pseudo dementia
E. Delirium
A

The correct answer is C.
Ms. Martinez has a normal mini mental status exam, her exam findings are normal with the exception of her affect. This suggests a mood disorder. **
As there is nothing in the history to suggest a recent loss, bereavement is not likely.
History is not suggestive of delirium which would present with more fluctuations in symptoms and alteration in cognition

67
Q

Ms. Brady, a 78-year-old female prevents to your office after six months for follow-up. Her interval history is significant for a TIA three months ago. Today her MMSE is 19. You note that six months ago her MMSE was 22 and nine months ago it was 26. Physical exam shows temperature of 98.8 Fahrenheit, blood pressure of 167/95 mmHg, heart rate of 76 beats/minute, and respiratory rate of 14 breaths/minute. Chest: regular rate and rhythm, no murmurs. Lungs: clear to auscultation. Neuro: weakness in the right upper extremity. Abdomen: soft, non-tender. She takes atorvastatin and aspirin. Lab studies show Hgb A1c: 5, TSH: 3, B12: 500 pg/mL.
Which of the following is the most important recommendation to prevent further disability in this patient?

A. Nothing can be done
B. Start metformin
C. Start vegetarian diet
D. Weight loss
E. Start hydrochlorothiazide
A

The correct answer is E.
To prevent additional TIAs and stroke, her blood pressure needs to be controlled. Hydrochlorothiazide is a reasonable first line medication.
Her HgbA1C is in the normal range so metformin would not be an appropriate choice.
A vegetarian diet would have little impact on limiting dementia, and we don’t know what she weighs and whether weight loss would be appropriate.

68
Q

A 68-year-old male becomes confused and agitated on post-operative day three after open cholecystectomy. He does not recall his surgery or where he is and wants to leave the hospital. Physical exam shows temperature of 99.9 Fahrenheit, blood pressure of 143/89 mmHg, heart rate of 90 beats/minute, respiratory rate of 13 breaths/minute, and O2 sat of 98% on room air. He appears agitated and uncomfortable. He has a Foley catheter. His physical exam is unremarkable. Labs: WBC-11,000, Urinalysis: 2+ nitrites and 2+ leukocyte esterase, 10 WBC/hpf. Glucose finger stick: 80, EKG: normal sinus rhythm at 88 beats per minute.
Which of the following is the best initial step in the management of this patient?

A. Remove urine catheter
B. Give IV ceftriaxone
C. Order urine gram stain
D. Order urine culture
E. Give oral ceftriaxone
A

The correct answer is A.
This patient has an UTI which is the most common cause of delirium in the elderly. Catheters increase the incidence of UTIs and removal would be the appropriate first step.
The other options would be appropriate to consider once the catheter is removed.

69
Q

Ms. Michaels is an 80-year-old female with a past medical history of shingles. She comes to your office accompanied by her daughter Jennifer who reports that her mother is forgetting things. Jennifer explains that her mother will ask the same question several times throughout the day although it was answered. Ms. Michaels also gets confused easily and is more passive than usual. Her memory problem was noticed two years ago after she forgot to pay her bills on multiple occasions. Jennifer now pays her mothers’ bills and cleans and cooks for her. Ms. Michaels’ vital signs are temperature of 99.2 Fahrenheit, blood pressure of 118/70 mmHg, heart rate of 80 beats/minute, and respiratory rate of 12 breaths/minute. Her physical exam is significant for bilateral osteoarthritis hand deformities. CT head shows mild atrophy of the hippocampus. Her MMSE is 20.
The patient’s diagnosis is most likely associated with?

A. Lewy bodies
B. Caudate nucleus atrophy
C. Plaque formation
D. Prion protein
E. Vascular disease
A

The correct answer is C.
Ms. Michaels symptoms and CT results are consistent with Alzheimer’s Disease as are plaque formations.
Lewy bodies are abnormal aggregates of protein that develop inside nerve cells in Parkinson’s disease.
Huntington’s disease presents with caudate nucleus atrophy.
Prion proteins are seen in transmissible spongiform encephalopathies

70
Q

Dr. Rodriquez, your family medicine preceptor, is giving a talk about ways to prevent delirium in patients during their hospital stay. What did she most likely suggest?

A. Only allow the patients visitors to come at night
B. Give diazepam every six hours
C. Keep the patient in restraints
D. Keep the patients room quiet
E. Keep the patients room well lit
A

The correct answer is E.
Stimulation such as a well lit room with activity has been shown to decrease delirium in hospitals.
Limiting visitors and keeping the room quiet could increase the risk of delirium.
Benzodiazepines such as diazepam can bring on delirium.
Restraints would be a response

71
Q

A 13-year-old female patient comes to your office for a physical. Her mother is concerned because she complains of menstrual cramps during her period each month. You determine that menarche was earlier that year and her periods have been mostly regular since that time. The pain is in her lower abdomen and is relieved with Ibuprofen and a heat pack. She has no other medical problems and her physical exam is normal.
What best describes this patient’s condition?

A. Premenstrual dysphoric disorder
B. Primary dysmenorrhea
C. Premenstrual syndrome
D. Secondary dysmenorrhea

A

The correct answer is B.
Primary dysmenorrhea is defined as the onset of painful menses without pelvic pathology, and is the most likely diagnosis in this case, given the normal history and physical.
Secondary dysmenorrhea implies painful menses secondary to some pelvic pathology, by definition.
Finally, this patient’s symptoms are during menses, as opposed to during the other phases of the menstrual cycle.

72
Q

A 13-year-old female patient comes to your office for a physical. Her mother is concerned because she complains of menstrual cramps during her period each month. You determine that menarche was earlier that year and her periods have been mostly regular since that time. The pain is in her lower abdomen and is relieved with Ibuprofen and a heat pack. She has no other medical problems and her physical exam is normal.
What is the most appropriate treatment for this patient?

A. Start OCPs
B. Start an SSRI
C. Continue Ibuprofen and heat packs, return if worsening
D. Start Danazol

A

The correct answer is C.
Primary dysmenorrhea is associated with increasing amounts of prostaglandins. Therefore, NSAIDs are often the first line of treatment, rather than the other therapies mentioned, of which OCPs would be the most common first/second-line treatment in the appropriate candidate.
SSRIs may help with comorbidities, but is not a primary treatment for primary dysmenorrhea.
Danazol is an androgenic medication with progesterone effects. It lowers estrogen and inhibits ovulation. It is primarily used for conditions like endometriosis or premenstrual syndrome, which this patient doesn’t have, and its multiple androgenic side effects, including weight gain, suppressing high-density lipids, and hirsutism, limit its desirability among patients.

73
Q

A 23-year-old female patient comes to your office complaining of bothersome symptoms the week before her period each month. She reports that she has significant breast tenderness, is very irritable, and eats significantly more than she does at any other time during the month. Her coworkers notice the difference in her mood and it is beginning to affect her interactions with them. The symptoms resolve after her period. She has no other medical problems or significant past medical history. Physical exam is normal.
What is this patient’s most likely diagnosis?

A. Premenstrual syndrome
B. Primary dysmenorrhea
C. Premenstrual dysphoric disorder
D. Secondary dysmenorrhea

A

The correct answer is A.
Premenstrual syndrome occurs in the second half of a woman’s cycle. It has physical symptoms like bloating, fatigue, and breast tenderness. The patient also may have behavioral symptoms like irritability, easy crying, and eating more than the rest of the month. There has to be an effect on the woman’s life to count as premenstrual syndrome.
Premenstrual dysphoric disorder is more severe and those criteria are listed in the DSM-IV for psychiatry. These symptoms have to significantly impair a woman’s life.
Primary dysmenorrhea is defined as the onset of painful menses without pelvic pathology. Secondary dysmenorrhea implies painful menses secondary to some pelvic pathology, by definition. Neither of these situations is implicated as the etiology of this patient’s problems.

74
Q

A 23-year-old female patient comes to your office complaining of bothersome symptoms the week before her period each month. She reports that she has significant breast tenderness, is very irritable, and eats significantly more than she does at any other time during the month. Her coworkers notice the difference in her mood and it is beginning to affect her interactions with them. The symptoms resolve after her period. She has no other medical problems or significant past medical history. Physical exam is normal.
What is the most effective treatment for this patient’s condition?

A. Danazol
B. OCPs
C. Continuous SSRI treatment
D. Regular exercise

A

The correct answer is C.
Selective serotonin reuptake inhibitors during menses are an effective treatment of premenstrual syndrome, especially if severe or mood symptoms predominate. There are three effective regimens for SSRI use. One regimen is continuous daily treatment. Another is intermittent treatment, which is just as effective as daily treatment for decreasing both psychologic and physical symptoms. There are two types of intermittent treatment. One method is to start therapy 14 days prior to menses (luteal phase of cycle) and continue until menses starts. The second method is to start on the first day a woman has symptoms and continue until the start of menses or three days later. Many randomized trials have used fluoxetine and sertraline. Venlafaxine can be used as well. Lower doses are effective. If one medication does not work, another in the same class should be tried prior to considering the treatment a failure. Follow-up should occur after two to four cycles. Intermittent treatment is associated with fewer side effects and lower cost.
Danazol is an androgenic medication with progesterone effects. It lowers estrogen and inhibits ovulation. However, its multiple androgenic side effects, including weight gain, suppressing high density lipids, and hirsutism, limit its desirability among patients. GnRH agonists, such as leuprolide, are effective at treating premenstrual syndrome through ovulation inhibition. However, their anti-estrogen effects, including hot flashes and vaginal dryness, make these less popular.
Oral contraceptives are effective treatment for dysmenorrhea, anovulation, and in some cases menorrhagia. While not always effective for premenstrual syndrome, they are a good place to start. It would be appropriate to try this in a woman also needing birth control. The most favorable pill is the formulation containing ethinyl estradiol and drospirenone. One study demonstrates potential improved effectiveness by decreasing the placebo pills to four days from seven.

75
Q

A 29-year-old female presents to your office complaining of very heavy periods. These started about six months ago. She reports that her periods were always normal until six months ago when she started passing a significant number of clots each day. Her periods occur in a regular pattern but usually last more than eight days. She reports having to change a super absorbent pad every two to three hours. On pelvic exam, the uterus is small, non-tender, and has uniform, smooth contour. On physical exam, you note a moderately enlarged thyroid and dry skin.
What is her most likely diagnosis?

A. Metrorrhagia
B. Menorrhagia
C. Uterine leiomyoma
D. Cervical polyp

A

The correct answer is B.
Menorrhagia is a descriptive diagnosis that has at the core of its definition, an increased length (usually ≥ 7 days) and quantify of menses. Quantity of menses is very difficult to define precisely and is only one of the terms that can mean abnormal uterine bleeding. The absolute criterion for menorrhagia is blood loss of more than 80 milliliters. Some providers try to use pad or tampon count. However there is variability in the absorption of different pads and how much blood a woman has on the pad prior to changing. Asking about clots may help, but again not easy to quantify. In fact, many women either over- or under-estimate the blood loss.
A uterine leiomyoma can present as menorrhagia, but when uterine fibraoids do present with menorrhagia, they are much more likely present with an enlarged uterus.
Cervical polyps more commonly present with irregular bleeding, often after intercourse.
Metrorrhagia is irregular bleeding.

76
Q

Which of the following physical findings indicate risk factors for coronary heart disease? Select all that apply.

A. Xanthelasma
B. Carotid bruits
C. Roth's spots
D. Diminished peripheral pulses
E. Buffalo hump
F. Striae
G. Increased waist circumference
A

> The correct answers are A, B, D, G
TEACHING POINT
Physical Findings Indicating Risk for CHD
Numerous findings on physical exam can be clues to underlying risk factors for CHD.
Findings of hypercholesterolemia:
Plaques or nodules composed of lipid-laden histiocytes, called xanthelasma (on the eyelids) and xanthomas (on extensor tendons)
Findings of vascular disease:
Carotid bruits
Diminished peripheral pulses
Hypertension
Increased abdominal aortic size (the diameter of a normal abdominal aorta should be less than 2 cm)
Other findings indicating increased risk of CHD:
Increased waist circumference (> 40 inches in men, > 35 inches in women)
Increased waist-to-hip ratio (> 0.85 for women or > 0.90 for men)
Roth’s spots (C) are retinal hemorrhages with pale centers, typically seen in bacterial endocarditis.
Findings such as buffalo hump (E), striae (F), moon facies, and bruising can be seen in states of cortisol excess. Depressed tendon reflexes may be seen in hypothyroidism. These are secondary causes of obesity.

77
Q

Which of the following are components of the metabolic syndrome? Select all that apply.

A. Impaired fasting glucose
B. Increased blood pressure
C. Increased low-density lipoprotein (LDL) cholesterol
D. Increased BMI

A

> The correct answers are A, B
TEACHING POINT
Metabolic Syndrome
Definition
The National Cholesterol Education Program defines the metabolic syndrome as any three of the following five:
Fasting plasma glucose equal and above 100 mg/dL (or on medical therapy for hyperglycemia)
BP ≥ 130/85 mmHg (or on medical therapy for hypertension)
Triglycerides ≥ 150 mg/dL (or on medical therapy for hypertriglyceridemia)
High density lipoprotein (HDL) cholesterol < 40 mg/dL for men, < 50 mg/dL for women (or on medical therapy for low HDL cholesterol)
Abdominal obesity (waist circumference > 40” for men, > 35” for women)
Mechanism
Insulin resistance is believed to be the underlying mechanism of the metabolic syndrome. There is some controversy about the existence of a metabolic syndrome, as opposed to a collection of independent risk factors; however this distinction should not discourage appropriate risk factor management and risk reduction.
Treatment
Aggressive lifestyle modification (dietary modification, physical activity, weight loss, and smoking cessation) is first-line therapy for the metabolic syndrome. Medication is often indicated if lifestyle modification is unsuccessful.
Coexisting disorders
The metabolic syndrome is also associated with a higher risk of:
diabetes mellitus
hepatic steatosis
hepatocellular carcinoma
cholangiocarcinoma
chronic kidney disease
obstructive sleep apnea
hyperuricemia
gout

78
Q

Which laboratory tests would you like to obtain for Mr. James? Select all that apply.

A. Fasting glucose
B. Complete blood count
C. Renal function (BUN and creatinine)
D. Fasting lipid profile
E. Serum cortisol
F. None at this time
A

> The correct answers are A, D
TEACHING POINT
Obesity Evaluation for Co-Existing Risk Factors
Patients with obesity should be evaluated for co-existing risk factors, including diabetes and dyslipidemia.
Diabetes can be diagnosed with blood glucose or hemoglobin A1C:
1. Blood glucose
A fasting blood glucose ≥ 126 mg/dL (≥ 7.0 mmol/L) confirmed on repeat testing or
A random blood glucose ≥ 200 mg/dL (≥ 11.1 mmol/L) with symptoms of hyperglycemia
(A fasting blood glucose between 100 and 125 mg/dL (5.6 and 7.0 mmol/L) indicates pre-diabetes.)
2. Hemoglobin (Hb) AIC
6.5% = diabetes
5.7 - 6.4% = prediabetes
Recommendations for screening for lipid disorders vary:
2013 guidelines from the American Heart Association and the American College of Cardiology recommend measuring total and HDL cholesterol levels every 4 to 6 years in adults aged 20 to 79 without atherosclerotic cardiovascular disease.
The U.S. Preventive Services Task Force (USPSTF) strongly recommends screening:
men > 35 years old
women > 45 years old if they are at increased risk for coronary heart disease
The U.S. Preventive Services Task Force (USPSTF) recommends screening:
men 20 - 35 years old if they are at increased risk for heart disease
women 20 - 45 years old if they are at increased risk for heart disease
USPSTF makes no recommendations for or against screening younger men (20-35 years old) and women of any age who are not at increased risk of coronary heart disease.
Although exact screening guidelines vary, there is consensus that screening is important. Epidemiologic studies had consistently shown a strong relationship between levels of total and LDL cholesterol and the rate of new onset CHD.
Emphasis on cardiovascular risk reduction through identifying and treating lipid disorders has resulted in a significant decrease in the prevalence of elevated cholestrerol from 34% to 17% between 1960 and 2000.
You decide you should check Mr. James’ fasting glucose and cholesterol.
Other options:
Complete blood count (B) and renal function (C) do not need to be checked in this healthy, asymptomatic 45-year-old man. Some experts advocate measuring transaminases to screen for hepatic steatosis in obese patients. To date, there is no evidence of improved clinical outcomes from such screening.
Mr. James does not have any symptoms or physical findings suggestive of an endocrine disorder causing his obesity. If a disorder such as hypothyroidism or Cushing’s syndrome were suspected, additional laboratory evaluation would be required.

79
Q

Dr. Simon asks you, “If Mr. James did need medication, which lipid-lowering medication would you begin?” Choose the single best answer.

A. Bile acid sequestrant (e.g., cholestyramine)
B. Nicotinic acid
C. HMG-CoA reductase inhibitors (e.g., statins)
D. Fibric acid (e.g., gemfibrozil)
E. Ezetimibe
F. Fish oil supplement

A

> The correct answer is C
TEACHING POINT
Lipid-lowering Medications
Statins (HMG-CoA reductase inhibitors) are first-line therapy for most patients who require lipid-lowering therapy because they can:
stabilize existing plaques, decreasing the risk of plaque rupture and myocardial infarction
decrease CHD and all-cause mortality
decrease LDL by 18% to 55%
increase HDL by 5% to 15%
decrease triglycerides by 7% to 30%
Bile acid sequestrants, such as cholestyramine, have a more modest effect on lowering LDL and raising HDL but can cause an increase in triglycerides. Usage has been limited by severe gastrointestinal distress and constipation. According to ACC/AHA guidelines, a non-statin lipid-lowering drug may be considered for patients with untreated LDL cholesterol < 190 after maximum intensity statin therapy has been achieved. Although some of these agents have beneficial effects on HDL cholesterol and/or triglycerides, there is little data to support beginning therapy to treat low HDL cholesterol or mild to moderate hypertriglyceridemia to prevent ASCVD.
Nicotinic acid (niacin, or vitamin B3) has a more modest effect on LDL. However, it is the most effective agent to increase HDL (by 15% to 30%). It can also decrease triglycerides (by 20% to 50%). Studies have shown conflicting results regarding use of nicotinic acid to prevent ASCVD. Usage of niacin has been limited by symptomatic body flushing. This flushing can be reduced by taking aspirin before the niacin dose.
Fibric acid derivatives are first-line therapy for reducing triglycerides but have only a modest effect on reducing LDL. They can increase HDL-cholesterol by 10% to 20%.
Ezetimibe inhibits absorption of cholesterol at the intestinal brush border and increases cholesterol clearance. Although LDL decreases with ezetimibe, there is no clear evidence of benefit in prevention of ASCVD when used as monotherapy or in combination with a statin.
Fish oil supplements have been shown to decrease triglycerides by 25% to 30%. They also increase HDL cholesterol slightly. Small studies suggest some decrease in progression of atherosclerosis in patients taking high doses of fish oil.

80
Q

Which four non-pharmacologic interventions lower LDL cholesterol? Select all that apply.

A. Reduction of saturated fat in diet
B. Increased dietary fiber
C. Alcohol cessation
D. Increased physical activity
E. Weight loss
A

> The correct answers are A, B, D, E
TEACHING POINT
Non-pharmacologic interventions to lower LDL
Lifestyle changes should be first-line therapy for all patients with dyslipidemias, whether or not medication is being prescribed.
ACC/AHA recommends lifestyle modifications as a critical component in ASCVD risk reduction.
Diet
Category
Recommendation
Evidence Quality
All adults
Mediterranean-style diet (the DASH dietary pattern achieves this).

This diet is rich in:

Vegetables
Fresh fruits
Whole grains
Lean meats: poultry, pork, fish
Legumes: e.g., lentils
Non-tropical vegetable oils: e.g., olive oil
Tree nuts: pecans, cashews, etc. (not peanuts)
And low in:
Sugar-sweetened beverages
Sweets
Red meats
A (strong)
Those needing LDL lowering

(recommendations should be made

irrespective of whether the patient

has an indication for a statin)
Reduce percent of calories from saturated fat, aiming for a goal of 5-6% of calories from saturated fats. Saturated fats come from:

Animal fats (meat and dairy)
Some vegetable oils (particularly coconut and palm oils)
Reduce percent of calories from trans fat. These come from:

Partially hydrogenated oils
Oils used for deep frying
Vegetable shortenings
Many pre-packaged baked goods and chips
A (strong)

A (strong)
Those needing BP lowering

(unlike with cholesterol management,

if a patient can lower his BP with diet,

he may avoid the need for medications)
Reduce sodium intake

Limit sodium intake to 2,400 mg per day
Limiting of sodium to 1,500 mg/day further lowers BP
A (strong)

B (moderate)
Exercise
On the basis of moderate quality evidence, all adults are encouraged to engage in moderate-to-vigorous intensity physical activity 3 to 4 times per week for 40 minutes per session.

81
Q

What caloric deficit is needed to lose one pound of body weight? Choose the single best answer.

A. 1,500
B. 2,000
C. 2,500
D. 3,000
E. 3,500
F. 4,000
G. 4,500
A

> The correct answer is E
TEACHING POINT
Caloric Deficit
A patient needs to have approximately a 3500 calorie deficit to lose 1 pound of weight.
Patients who believe they are following these recommendations for caloric intake may not lose this much weight for several reasons:
1. Tendency to underestimate actual caloric intake.
Patients should be encouraged to read food packaging labels and, if necessary, weigh or measure their food to determine accurate portion sizes and caloric content.
2. These numbers are only estimates of the actual caloric requirement.
Although there is an activity factor in the calculation, variations in day-to-day physical activity and individual metabolic rates can result in substantial variation in energy requirements.
3. The “plateau” phenomenon.
As patients lose weight, their caloric requirements decrease and they eventually reach a point where they stop losing weight, even if they maintain the caloric intake previously producing weight loss.
Regardless of the cause, if patients are unable to lose weight or continue to gain weight, they need to further restrict calories or increase physical activity.

82
Q

Other than dietary modifications and exercise, which therapies are approved for management of obesity? Select all that apply.

A. Oral drug therapy
B. Liposuction
C. Insulin injection
D. Bariatric surgery
E. Laxative usage
A
> The correct answers are A, D
TEACHING POINT
Management of Obesity
Pharmacologic therapy and bariatric surgery are both approved for management of obesity. However, it is important to emphasize to patients that these are adjuncts to dietary changes and physical activity, not replacements for them.
Oral drug therapy
Indications: All weight loss medications approved by the FDA are indicated for patients with BMI > 30; most are also indicated for patients with BMI > 27 with at least one obesity-related comorbidity such as hypertension or diabetes.
Medication
Mechanism
Side Effects
Note
Phentermine

Diethylpropion

Pendimetrazine

Benzphetamine
Noradrenergic appetite suppressant
tachycardia
insomnia
headache
dizziness
anxiety
constipation
diarrhea
vomiting
Indicated for short-term (a few weeks) only
Orlistat
Gastrointestinal lipase inhibitor
Decreases fat absorption
gastrointestinal discomfort
fecal incontinence
malabsorption of fat-soluble vitamins
Available without a prescription
Lorcaserin
Highly selective serotonergic receptor agonist appetite suppressant
headache
dizziness
nausea
cough
constipation
Associated with hypoglycemia in diabetic patients
Qsymia (combination of phentermine and topiramate)
Acts via multiple pathways to suppress appetite
paresthesias
dizziness
insomnia
cognitive changes
tachycardia
constipation
Contraindicated in pregnancy
Contrave (naltrexone/bupropion)
Naltrexone is an opioid antagonist, and bupropion is a relatively weak inhibitor of the neuronal reuptake of dopamine and norepinephrine.
headache
sleep disorder
constipation
nausea
vomiting
hypertension
tachycardia
Not approved for use in the treatment of major depressive disorder, other psychiatric disorders, or smoking cessation. To be used with caution in patients with cardiovascular or liver disease.
Studies have shown that-when combined with lifestyle modifications-these medications result in an additional weight loss of 3% to 9% compared to placebo.
Bariatric surgery
Indications: Patients with BMI > 40 or BMI ≥ 35 with associated severe health complications who have not succeeded in losing weight with other treatment methods.
Procedures include:
Gastric bypass-partitioning of the stomach with attachment of the proximal stomach to the jejunum
Adjustable gastric banding-placement of an inflatable, adjustable band around the stomach
Vertical banded gastroplasty-partial partitioning of the proximal stomach with placement of a small, fixed-diameter gastric outlet stoma
Liposuction (B) is cosmetic surgery and is not designed for permanent weight loss.
Insulin (C) causes weight gain, not weight loss.
Laxative use (E) is ineffective for long-term weight loss and can cause electrolyte disturbances.